Community Exam 3 **

Ace your homework & exams now with Quizwiz!

C

A school nurse was caring for an 8-year-old child who had been hurt on the school playground. During the nurse's assessment, the child admitted that her mother was working but didn't make much money so the girl and her mother were living in their car. Based on this information, which of the following would the nurse most likely suspect? a. The child is accident-prone and clumsy. b. The child is being bullied and pushed around by other children. c. The child is more likely to have school absences. d. The child is in need of a vision screening.

D

A student engages in unprotected sex under the influence of alcohol. The student decides to have an HIV test completed the next day. Which of the following results will most likely occur? a. The results will probably be negative for HIV. b. The results will probably be positive for HIV. c. The probability of disease is so low there is no reason to be tested. d. The test results won't be reliable so soon after exposure.

C

A teacher asks the school health nurse to assess a child for neglect. Which of the following assessment findings could indicate neglect? a. Bruises in various stages of healing b. Failure of parent to attend parent-teacher conferences or return teacher phone calls c. Lack of weight gain and wearing dirty clothes d. Lice in the hair

29. What factors result in rural residents having less access to health care than urban dwellers? Select all that apply. a. Employment opportunities may be low paying and seasonal. b. Greater distances to travel and limited choice of health care providers c. Health care providers prefer to work in higher-paying urban settings. d. Rural residents are less likely to have health insurance. e. Rural persons' economic resources are tied up in land, not easily accessible cash. f. Rural persons believe in self-sufficiency and self-care, not asking others for help.

A, B, C, D

ABC

A mother is concerned that her daughter may be considering experimentation with drugs and alcohol. Which of the following strategies would the nurse suggest the mother implement? (Select all that apply.) a. Provide opportunities for her daughter to develop her talents. b. Provide realistic feedback to the daughter about her performance at school. c. Encourage her daughter to become involved in extracurricular activities. d. Encourage her daughter to develop new friendships.

C

A movie shows a woman and a man having a contest to see who can drink more shots of whiskey. Who is more likely to "win" the drinking contest when the other passes out and cannot continue? a. If their drinks are the same size, they'll both pass out about the same time. b. If they play fair, they'll probably both vomit before passing out. c. The man will win. d. The woman will win.

A B C D E

A multisystem approach to community disease control would include such interventions as: (select all that apply) Select all that apply. a. community action programs. b. control of vectors. c. improved surveillance systems. d. legislation. e. provision of chemotherapy.

c

A new role for public health that is used in widespread public emergencies is: a. communicable disease control. b. first responder. c. incident commander. d. professional triage.

A

A nurse at a migrant clinic assists clients with completing the application process to receive supplemental food assistance. Which of the following roles of the nurse is being used? A. Advocate B. Case manager C. Outreach worker D. Primary caregiver

A

A nurse in community health is following a pregnant teenager who attends school. The nurse plans to discuss self-care activities that will be important for the teen during her pregnancy. The discussion should include: A. carrying heavy book bags. B. changing to home education. C. decreasing fluid intake to avoid nausea. D. keeping up her grades.

B

A nurse is considering applying for a public health nurse position at a local health department. Which of the following minimum levels of educational preparation should the nurse have obtained? A. Associate degree in nursing B. Baccalaureate degree in nursing C. Master of science degree in nursing D. Training as a licensed practical nurse

D

A nurse is counseling a woman following a rape. Which of the following feelings is the woman most likely to state she felt while being raped? a. Confusion b. Satisfaction c. Sexual release d. Helplessness

AB

A nurse is helping a client's family prepare to have an intervention that hopefully will encourage the alcoholic father to realize the need for change. Which of the following instructions by the nurse to the client's family members would be most effective? (Select all that apply.) a. "Express your love and your belief that he can change." b. "Point out that his behavior is his choice and his responsibility." c. "Let him know that you understand how much he loves being with his drinking pals and that you agree that their friendship is important." d. "Remind him that entering treatment is better than going to prison."

C

A nurse is implementing a primary prevention strategy to address abuse in the community. Which of the following actions is the nurse most likely taking? a. Encouraging others to interfere when they see children hit or hurt in a public place b. Referring caregivers to community respite centers if abuse is beginning to occur c. Lobbying for passage of legislation to outlaw physical punishment in schools d. Screening each pregnant woman privately for intimate partner abuse

C

A nurse is trying to decrease the risk for depression among older adults. Which of the following interventions should the nurse implement? a. Encourage moving to a nursing home where others of the same age reside. b. Monitor for signs and symptoms of depression. c. Organize an exercise and health promotion program at the local senior citizen center. d. Teach older clients to focus on their strengths rather than their weaknesses.

B

A nurse is working to ensure long-term positive health outcomes of poor pregnant teens and their children. Which of the following actions would the nurse most likely take? a. Assist teen mothers to learn about their body changes during pregnancy. b. Develop programs that allow teen mothers to complete their education. c. Offer courses in proper care of babies and how to be a parent. d. Monitor pregnant teens to detect early problems with pregnancy.

C

A nurse is working to improve her self-awareness in order to provide better client-centered care to clients living in poverty. Which of the following questions should the nurse ask herself? a. What would I do if I lived in poverty? b. What can be done to get poor people motivated to work? c. How have the media images of poor persons shaped my image of poverty? d. How can community resources assist those living in poverty?

1. When determining whether a geographic area is rural or urban, the nurse should recognize that: A. rural and urban areas, by relative nature, occur on a continuum. B. rural regions have fewer than six persons per square mile. C. rural residents feel isolated. D. rural areas are recreational, retirement, or resort communities.

A

5. A nurse in community health located in Virginia is conducting an assessment on a Hispanic worker currently working in a local apple orchard for the season. The nurse determines that the worker originates from Florida and is living in temporary housing with other orchard workers. Based on this information, the nurse should integrate the special needs of what vulnerable population? A. Migrant farmworkers B. Seasonal farmworkers C. Underinsured D. Undocumented aliens

A

7. What have surveys of public health experts determined to be the number one priority in rural areas? a. Access to health care b. Access to education via computer and Internet c. Access to Internet including the World Wide Web d. More and better-paying employment opportunities e. More up-to-date energy-efficient housing

A

d

A current dramatic trend in nursing in community health involves: a. clinical surveillance. b. shifts in research focus. c. special needs programs. d. vulnerable population shifts.

When determining whether a geographic area is rural or urban, the nurse should recognize that: A. rural and urban areas, by relative nature, occur on a continuum. B. rural regions have fewer than six persons per square mile. C. rural residents feel isolated. D. rural areas are recreational, retirement, or resort communities.

A. rural and urban areas, by relative nature, occur on a continuum.

9. A nurse says, Im not going to change the way I practice nursing based on where the client is from, because research shows that Western health care technology and research is best. Which of the following is being demonstrated by the nurses statement? a. Ethnocentrism b. Prejudice c. Racism d. Stereotyping

ANS: A Ethnocentrism, a type of cultural prejudice at the cultural population level, is the belief that ones own group determines the standards for behavior by which all other groups are to be judged. For example, some American nurses and providers may think, The way we do it is the only right way to provide this care.

32. A nurse lobbies with legislators for the passage of legislation to prohibit smoking of tobacco products in public places. Which level of prevention is being implemented? a. Primary b. Secondary c. Tertiary d. Health promotion

ANS: A When using primary prevention, the focus is on health promotion and disease prevention.

1. Which of the following sexually transmitted diseases can be prevented through immunization? a. Chlamydia b. Gonorrhea c. Hepatitis B d. Herpes

ANS: C At the time of this writing, HBV infection can be prevented by immunization; vaccines for the remaining options are not yet available.

13. A client is crying softly and saying, What did I do to deserve this punishment, Lord? Which of the following responses by the nurse would be the most appropriate? a. God doesnt punish people. Youre sick just because of bad luck. b. I can call the hospital chaplain to help you talk about these feelings c. What can I do to be helpful to you right now? d. Would you like to confess your sins and repent so this illness will go away?

ANS: C Some clients may view their illness as punishment for misdeeds and may have difficulty accepting care from nurses who do not share their beliefs. Because the nurse may not be a member of the clients religious faith group, an open-ended response showing caring is the most appropriate statement.

11. Which statement about nicotine and smoking is accurate? a. Cigars contain higher doses of nicotine than cigarettes. b. Tolerance to nicotine takes days to develop. c. Sidestream smoke contains more toxic compounds than mainstream smoke. d. The harsher smoke of pipes and cigars is more hazardous than cigarette smoke.

ANS: C It is true that sidestream smoke contains greater concentrations of toxic and carcinogenic compounds than mainstream smoke.

8. The oldest and most widely used psychoactive drug in the world is: a. Marijuana b. Heroin c. Alcohol d. Cocaine

ANS: C The oldest and most widely used psychoactive drug in the world is alcohol.

Alcoholism

Addiction to alcohol

B

All adults should be assessed for violence in their primary intimate relationships. The abuse of female partners has the most serious community health ramifications because of the greater prevalence, the more serious long-term emotional and physical consequences, and the greater potential for: A. fleeing to a shelter. B. homicide. C. possessive behavior. D. spontaneous abortion.

Harm reduction

Also called harm minimization; a public health approach to substance abuse problems. This approach acknowledges, without judgment, that licit and illicit drug use is a reality, and the focus of interventions is to minimize these drugs' harmful effects rather than to simply ignore or condemn them; also to facilitate responsible use of substances.

A nurse in community health conducting a home visit notices a 4-year-old girl sitting on a stool in an adjoining room. The girl is quiet and withdrawn, rarely makes eye contact, and does not leave the room. The nurse proceeds to ask about the child and attempts to engage the child in conversation. The nurse is assessing for what indicators of child abuse? A. Emotional abuse B. Emotional neglect C. Physical abuse D. Physical neglect

B. Emotional neglect

2. A nurse in community health is working in a rural setting. In planning for programs to address the population's needs, the nurse should be aware that, in general, rural populations: A. engage in physical activity during leisure time. B. engage in preventive behaviors. C. perceive their overall health as less favorable. D. use seat belts.

C

9. A migrant farmworker presents to the clinic reporting an acute onset of severe abdominal pain, nausea, vomiting, diarrhea, and headache with difficulty concentrating. What condition might cause such symptoms? a. Appendicitis b. Bacterial gastroenteritis c. Pesticide poisoning d. Viral illness

C

An undocumented migrant farmworker has been diagnosed with tuberculosis (TB). The local health department initiates treatment by dispensing the first month's supply of medication and educates the client on the need to continue treatment for 6 to 12 months. A major challenge that the client may face related to ongoing treatment for TB is: A. affordable care. B. discrimination. C. fragmented services. D. language barriers.

C. fragmented services.

A community-based HIV/AIDS clinic would be concerned about which aspects of the Public Health Code of Ethics?

Confidentiality when possible. Promoting advocacy for disenfranchised persons.

12. What are the two primary causes of death in children who live in rural areas? a. Accidents and genetic conditions b. Accidents and cancer c. Homicide and suicide d. Machinery-related accidents and drowning

D

state public health agency

Each of the U.S. states and territories has a single identi ed of cial state public health agency, managed by a state health commissioner

Continuous Quality Improvement

Internal determinants Determines requirements and deficiencies and expectations Identifies process improvement opportunities Prevention All members in the organization are responsible Inspires hope.

Brief interventions

Interventions that are sometimes made by health care professionals who are not treatment experts and that have been found to be effective in helping alcohol, tobacco, and other drug abusers and persons with addictions reduce their consumption or follow through with treatment referrals. They can have six parts: feedback, responsibility, advice, menu of options, empathy, and self-efficacy.

case manager

Nurse who works to enhance continuity and provide appropriate care for clients whose health problems are actually or potentially chronic and complex. School nurse who performs general activities concerning health problems of the children. Builds on the basic functions of the traditional role and adapts new competencies for managing the transition from one part of the system to another or to home

Level of Prevention-rabies post exposure immunization

Secondary Prevention

Planning

Selecting and carrying out of a series of actions to achieve stated goals

Hepatitis B Virus

Spread through blood and body fluids; can survive at room temperature for at least 1 week

Hepatitis C Virus

Spread through blood or body fluids Most common chronic bloodborne infection in the U.S.; leading cause of chronic liver disease, end-stage liver disease, liver cancer, and liver transplants in the U.S.

ACD

Which of the following activities are addressed by most public health departments on the federal, state, and local levels (Select all that apply.) A. Collecting and analyzing various health statistics B. Fining or terminating business at any facility that engages in unsafe commercial practices C. Identifying and planning for high-risk populations D. Planning for and responding to emergencies

C

Which of the following is the top risk factor for intimate partner homicide? a. There is a gun in the house. b. The husband has made threats against wife before. c. The wife has previously called 911 because of husband's beating her. d. The woman's young daughter also lives with them.

B

Which of the following racial groups experiences the highest incidence of poverty? a. Non-Hispanic Whites b. Blacks c. Hispanics d. Asians

C D

Which of the following symptoms suggests smallpox as opposed to the more common and much less dangerous chickenpox? (Select all that apply.) a. Child appears only mildly ill until late stages in smallpox. b. Lesions appear in various stages in the same area of the body rather than all at once. c. Rash lesions are most abundant on the face and extremities, not on the trunk. d. Rash occurs 2 to 4 days after sudden onset of fever rather than with the fever.

severe acute respiratory syndrome

a previously unknown (prior to 2003) disease of undetermined etiology and no definitive treatment emerged in China, Hong Kong, Taiwan, Singapore, and Canada--only a few cases have been reported since 2003

virulence

ability to produce a severe pathological reaction

Blood alcohol concentration (BAC)

also called blood alcohol level (BAL); the amount of alcohol in the blood, commonly expressed as grams of alcohol per 100 milliliters of blood. Most state legal limits of intoxication while driving are 0.08% or 0.1%.

agent

an infectious agent can cause infection or disease

four main categories of infectious agents

bacteria, fungi, parasites, and viruses

vectors

can be arthropods such as ticks and mosquitoes or other invertebrates such as snails that can transmit the infectious agent by biting or depositing the infective material near the host

A nurse in community health is following a pregnant teenager who attends school. The nurse plans to discuss self-care activities that will be important for the teen during her pregnancy. The discussion should include: carrying heavy book bags. changing to home education. decreasing fluid intake to avoid nausea. keeping up her grades.

carrying heavy book bags. If teens return to school, it is important for the nurse to discuss these needs: (1) using the bathroom frequently, (2) carrying and drinking more fluids or snacks to relieve nausea, (3) climbing stairs and carrying heavy book bags, and (4) fitting comfortably behind stationary desks.DIF: Cognitive Level: ApplicationREF: Page 421

Virtue ethics is distinctly different from moral justification via theories or principles because the emphasis of virtue ethics is practical reasoning applied to:

character development.

Identification of the discrepancies between the quality standards of the agency and the actual practice of the health care professionals is part of the interpretation component of quality assurance programs. Other factors addressed during this stage are: (select all that apply)

choices of possible courses of action. strengths and weaknesses.

Quality improvement strategies

consistently providing appropriate and effective care. Reducing unjustified geographic variation in care Eliminating avoidable mistakes Lowering access barriers Improving responsiveness to client Eliminating racial, ethnic, gender socioeconomic, and other disparities and inequalities in access and treatment.

A general approach to quality improvement where a qualified agent designates formal recognition to individuals or institutions that have met minimum standards of performance is best described as:

credentialing.

Quality Health Care

defined by public health as "the degree to which health services for individuals and populations increase the likelihood of desired health outcomes and are consistent with current professional knowledge."

Quality

defined by the client as the improvement in health status

Quality

defined by the organizations and providers as accurate intervention, the clinical context of care, and the skill of the provider.

In caring for a young adult from West Africa, the community nurse is introduced to another individual who is referred to as "auntie." A culturally competent nurse who is aware of the basic organizing factor of culture related to social organization would:

find out who is considered to be a member of the family.

A nurse who speaks only English has just gotten a new client for an intake interview. The client is a refugee who has very limited English proficiency (LEP). The nurse should:

get an interpreter.

The nurse in community health is meeting with staff to systematically plan for a new outreach program. Doing so helps them to:

identify the resources and activities that will help them meet their program objectives.

unauthorized immigrant

illegal alien/undocumented; only eligible for emergency medical services, vx, tx for communicable diseases, school lunches

cultural knowledge

info about organizational elements of diverse cultures and ethnic groups; learn about pts worldview from epic perspective

legal immigrant

lawful permanent resident; able to live and work in U.S.

Uncomplicated gonorrhea

limited cervical or urethral infection.

A nurse in community health working in an inner city clinic with high poverty and unemployment rates recognizes the need for programs for pregnant women because these women often receive late or no prenatal care and deliver: at home. full term. identical twins or triplets. low-birthweight babies.

low-birthweight babies. Poor pregnant women are more likely than other women to receive late or no prenatal care and to deliver low-birthweight babies, premature babies, or babies with birth defects.DIF: Cognitive Level: KnowledgeREF: Page 411

infectiousness

measure of the potential ability of an infected host to transmit the infection to other hosts.

Goal of Primary Prevention related to infectious disease

prevent occurrence of the disease--prevent it before it happens

Goal of Secondary Prevention related to infectious disease

prevent the spread of disease once it occurs--activities center around rapid identification of potential contacts to a reported case.

Herpes simplex virus 1 (HSV-1)

primarily causes nongenital lesions, such as cold sores, that may appear on the lip or mouth.

environment

refers to the physical, biological, social, and cultural factors that are external to the human host. They can facilitate the transmission of an infectious agent from an infected host to other susceptible hosts.Reductions in communicable disease risk can be achieved by altering these environmental factors. (Think mosquito control, sewage systems, washing utensils after using them on raw meat.)

Worldwide Leading Causes of Death from Infectious Diseases

respiratory infections, diarrheal diseases, HIV/AIDS, TB, malaria, meningitis, pertussis, measles, hep B, plus more...

ethnicity

shared feelings of peoplehood reflecting cultural membership based on sharing similar cultural patterns that overtime create common hx resistant to change; reciprocal relationship between individual and society

surveillance system

systematic collection, organization, and analysis of current, accurate, and complete data for a defined disease condition

A nurse in community health seeks to determine whether the visit with an older adult Asian woman is successful. The nurse has not encountered many clients from this culture. An effective way to judge whether a cultural encounter has been effective would include the nurse's sense that the visit was successful, the nurse and client experience little or no stress, and:

tasks are done efficiently.

infectivity

the ability to enter and multiply in a host

Congenital syphilis

transmitted transplacentally and, if untreated, can cause premature stillbirth, blindness, deafness, facial abnormalities, crippling, or death.

ethnocentrism

belief that one's own group determines the standards for behavior by which all other groups are to be judged; cultural prejudice at population level; may think the way you do something is the best way

racism

belief that persons who are born into a particular group are inferior; a form of prejudice

The deliberate release of viruses, bacteria or other germs with the intent of causing illness or death is:

biological terrorism.

Tracer model/Method

type of model to evaluate quality Is a measure of both process and outcome of care and is more effective in evaluating the health care of groups than of individuals clients. it is also more effective in evaluating care delivered by an institution than care delivered by an individual provider.

Forms of disease prevention

vaccine administration, early detection, or teaching clients about abstinence or safer sex.

Assault

violent physical or verbal attack.

A current dramatic trend in nursing in community health involves:

vulnerable population shifts.

Mental health

Ability to engage in productive activities and positive relationships and to adapt to change and cope with adversity.

virtue ethics

Asks "What kind of person should I be?" and purports that people should be allowed to flourish as human beings

sexual debut

First intercourse

Progress

Formative or evaluation of processes

PATCH goal

Health education model

Tolerance

In pharmacology, the need for increasing doses of a drug over time to maintain the same effect.

Poysubstance use or abuse

Drugs from different categories used together or at different times to regulate how the person feels.

Psychoactive Drugs

Drugs that affect mood, perception, and thought.

Second-hand smoke or side stream smoke

Drugs that affect mood, perception, and thought.

role model

Person who is an example of professional or personal behavior for others

PATCH

Planning approach to community heath

Stimulants

Drugs that increase the activity of the central nervous system, causing wakefulness.

Depressants

Drugs that reduce the activity of the central nervous system.

Child abuse

active forms of maltreatment of children

A major emerging public health issue facing nursing in community health is:

welfare reform.

Suburbs

Areas adjacent to a highly populated city.

Consumer price index (CPI)

Basic indicator of inflation—a measurement of inflation by comparison of prices overall and of categories of consumed goods and services purchased by urban wage earners and their families over a certain period.

six characteristics of an infectious agent

1 Infectivity 2 Pathogenicity 3 Virulence 4 Toxicity 5 Invasiveness 6 Antigenicity

Which of the following systems of surveillance is used to monitor trends in commonly occurring diseases?

Sentinel

A

Society bears the burden of infectious disease. That burden includes the effects of morbidity and mortality as well as the staggering: A. economic burden. B. fear burden. C. incidence burden. D. vulnerability burden.

Multi system Approach to Communicable Disease Control

1. Improve host resistance (hygiene, nutrition, fitness, immunization, stress, and mental health) 2. Improve safety of the environment (sanitation, safe water, clean air, proper cooking and food storage, vector control, and control of animal reservoir hosts) 3. Improve public health systems (increased access to health care, health education, improved surveillance systems) 4. facilitate social and political changes to ensure better health for all (individual, organizational, and community action, legislation)

Ways to Prevent Infectious Disease at Home

1. Wash your hands. 2. Clean and disinfect surfaces. 3. Handle and prepare food safely 4. Get immunized. 5. Use antibiotics appropriately. 6. Be careful with pets. 7. Avoid contact with wild animals

B

Which of the following approaches has research suggested is the most effective way to approach drug addiction? a. Criminal justice system b. Harm reduction c. Punishment approach d. Substance abuse education

The nurse must ask about violence at each prenatal and postpartum visit, especially with vulnerable populations such as teenagers, as well as observe for signs of violence on each visit. The nurse should be aware that the peak for postpartum intimate partner violence may be observed at various times according to ethnic group, such as: 6 months for white mothers. 10 months for white mothers. 3 months for African-American and Hispanic/Latino mothers. 12 months for African-American and Hispanic/Latino mothers.

3 months for African-American and Hispanic/Latino mothers. Violence that begins in pregnancy may continue for several years after, with increasing severity. Variations by ethnicity have also been observed during this postpartum period: intimate partner violence may peak at 3 months postpartum among African-American and Hispanic/Latino new mothers and at 18 months for white mothers. The nurse should look for physical signs of abuse, as well as for controlling or intrusive partner behavior.DIF: Cognitive Level: ComprehensionREF: Page 419

community health

Meeting collective needs by identifying problems and managing interactions within the community and larger society. The goal of community-oriented practice

B

A district health nurse is assigned to two rural counties in the state. To achieve the best outcomes possible in reducing the health disparities for the large number of frail older clients in the two counties, the nurse should consider using what community-oriented nursing skill? A. Assessment B. Case management C. Geriatrics D. Tertiary prevention

A

A nurse would like to increase the accessibility to health care services for mentally ill homeless clients in the community. Which of the following actions should be taken by the nurse? a. Apply for a grant to fund a mobile clinic to take health care to the clients. b. Distribute flyers to the homeless that detail the location of various types of health care services. c. Refer homeless clients to areas that provide temporary housing. d. Solicit donations for food and clothing to be distributed to the homeless.

A

A nurse's Mantoux test is positive for exposure to tuberculosis. Which of the following conclusions should be drawn by the nurse? a. The nurse has been exposed to tuberculosis. b. The nurse has tuberculosis. c. The positive test result probably is due to a problem in the testing process. d. The test is inaccurate and needs to be repeated.

Point Source Outbreak

A pattern of occurrence in which all persons exposed become ill at the same time, during one incubation period.

D

A public health nurse has reviewed the objectives of Healthy People 2020. Which of the following should be the focus for programming in the community related to communicable diseases? A. Encouraging community members to engage in healthy behaviors B. Identifying high-risk sexual behaviors among community members C. Monitoring the sexual activity of adolescents D. Providing information about the hazards of multiple sexual partners

A

A resident in a rural community has been diagnosed with asthma. Which of the following providers will most likely provide care to this client? a. Advanced nurse practitioner b. Allergist c. Pediatrician d. Pulmonologist

B

A school health nurse is asked by a parent group to explain risk factors for alcoholism. Which of the following information should the nurse include? a. Alcoholism is determined primarily by the family environment. b. Alcoholism is determined partly by genetic factors. c. Persons who are alcoholics are usually women. d. Persons born with fetal alcohol syndrome are alcoholics from birth.

Very little is known about quality of care in the U.S. Because:

A variety of definitions of quality are used It is difficult to get comparable data from all providers and health care agencies

A recent movie release portrays a criminal as a black female drug user whose abusive boyfriend has two children by different women. She lives in the riot-torn inner city of a large metropolitan area. This best demonstrates what type of factor that influences poverty? A. Cultural B. Environmental C. Political D. Social

A. Cultural

A nurse in community health located in Virginia is conducting an assessment on a Hispanic worker currently working in a local apple orchard for the season. The nurse determines that the worker originates from Florida and is living in temporary housing with other orchard workers. Based on this information, the nurse should integrate the special needs of what vulnerable population? A. Migrant farmworkers B. Seasonal farmworkers C. Underinsured D. Undocumented aliens

A. Migrant farmworkers

A nurse in community health is working with a parent whose spouse has been called up for active duty in the military reserve. The family is experiencing financial strain due to decreased income. The extended family lives at a distance. The parent is struggling to manage the family in the spouse's absence. The family consists of four children (three preschool and one preteen). In this situation, it would be important for the nurse to further explore the potential for: A. child abuse. B. depression. C. intimate partner abuse. D. parent's resentment of the preteen.

A. child abuse.

In giving care to the survivors of violence, the nurse should demonstrate respect and caring for all family members, insist that safety is the first priority, and demonstrate intolerance for violent behavior. Additionally, the nurse should be: A. absolutely honest about what will be reported and what the family can expect. B. authoritarian in approaching the problem. C. cautious in reporting unconfirmed reports of violence. D. sincere in concern for the victims.

A. absolutely honest about what will be reported and what the family can expect.

A nurse in community health is following a pregnant teenager who attends school. The nurse plans to discuss self-care activities that will be important for the teen during her pregnancy. The discussion should include: A. carrying heavy book bags. B. changing to home education. C. decreasing fluid intake to avoid nausea. D. keeping up her grades.

A. carrying heavy book bags.

Summarize evaluation

Assess program outcomes or are a follow up of the results of the program activities and look at end result

24. When implementing the harm reduction model, the nurse focuses on using which approach? a. Education b. Law enforcement c. Scare tactics d. Rehabilitation

ANS: A The harm reduction model accepts the reality that psychoactive drug use is endemic, and it focuses on pragmatic interventions, especially education, to reduce the adverse consequences of drug abuse and get treatment for addicts.

6. Which factor has contributed to the problem of substance abuse? a. Increased knowledge about the use of drugs b. Social acceptability of certain drugs c. Recognition of substance abuse as a health problem d. Control of the content and strength of illegal drugs

ANS: B Social acceptance of certain drugs contributes to the substance abuse problem.

3. The use of any substance that threatens a persons health or impairs his or her social or economic functioning is: a. Drug dependence b. Substance abuse c. Drug addiction d. Illegal

ANS: B Substance abuse is the use of any substance that threatens a persons health or impairs his or her social or economic functioning.

9. How much alcohol can the liver metabolize in an hour? a. 1 ounce of whiskey b. 3 ounces of wine c. 5 ounces of wine d. 12 ounces of beer

ANS: D Whiskey is 3/4 oz and wine is 4 oz.

Hallucinogens

Also known as psychedelics; drugs that stimulate the nervous system and produce varied changes in perception and mood.

A

An American takes a long-awaited vacation in sunny Mexico, spending days on the beach eating fresh raspberries from a nearby vendor and drinking bottled water. The tourist may be altering: a. agent-host-environment interaction. b. circadian rhythms. c. herd immunity. d. resistance.

cultural repatterning

RN works with pts to help them reorder, or modify their cultural practices when practices ARE harmful to them; Ex: these herbs cause HTN

Continuous quality improvement (CQI)

An approach to managing quality that emphasizes continual improvement in real time, empowering employees to manage quality themselves, and making changes in organizational systems to enable workers to provide high-quality services.

B

An eighth-grader is brought to the emergency department by a parent for unusual skin blistering and discoloration around the nose and lips. The parent states that the child will not say what caused the injury. The nurse should consider the possible use of: A. crystallized methamphetamine. B. inhalants. C. MDMA (Ecstasy). D. PCP (phencyclidine).

C

An example of secondary prevention for infectious disease prevention is: A. malaria chemoprophylaxis. B. Pneumocystis carinii pneumonia (PCP) chemoprophylaxis for people with AIDS C. quarantine. D. restaurant inspections.

C

An instructor is reviewing Salmonella infections with her class. Which of the following comments indicates that the student needs further review on how Salmonella is spread? a. "Certain pets and farm animals may be Salmonella carriers." b. "It is possible to transmit Salmonella by person-to-person contact." c. "Salmonella may be spread by spores that form once contaminated blood is exposed to the air." d. "Salmonella outbreaks are usually due to contaminated meat, poultry, and eggs."

b

As a result of recent social policy changes, public health agencies have shifted their emphasis from a focus on primary health care services to a focus on core public health activities because of new and re-emerging public health issues. A critical aspect of ensuring the public health system's ability to address these issues is: a. additional regulation. b. adequate funding. c. bioterrorism strategy. d. media coverage.

Program management process

Assessing, planning, implementing and evaluating a program

APEXPH

Assessment protocol for excellence in public health

When using the health measure of death rates for working-age adults, the nurse could expect to find the highest rates in which areas? A. Large metropolitan areas B. Most rural and highly populated urban areas C. Most rural and suburban areas D. Small suburban and all urban areas

B. Most rural and highly populated urban areas

When using the health measure of death rates for working-age adults, the nurse could expect to find the highest rates in which areas? A. Large metropolitan areas B. Most rural and highly populated urban areas C. Most rural and suburban areas D. Small suburban and all urban areas

B. Most rural and highly populated urban areas Death rates for working-age adults are higher in the most rural and the most highly populated urban areas. The highest death rates for children and young adults occur in the most rural counties. Residents of rural areas have the highest death rates resulting from unintentional injuries in general and motor vehicle injuries in particular.DIF: Cognitive Level: KnowledgeREF: Page 392

The nurse at the adult day care center notices bruises on the wrists of a 90-year-old client. Besides the physical assessment of the client, the nurse should: A. confront the daughter when she arrives to pick the father up. B. discuss the findings with the caregivers to determine the cause of the injuries. C. educate the staff about indications of elder abuse. D. make a referral to the primary care provider for follow-up.

B. discuss the findings with the caregivers to determine the cause of the injuries.

Violence is a major public health problem in our communities that causes premature mortality and lifelong disability. Violence-related morbidity is a significant factor in: A. community deterioration. B. health care costs. C. juvenile delinquency. D. population density.

B. health care costs.

All adults should be assessed for violence in their primary intimate relationships. The abuse of female partners has the most serious community health ramifications because of the greater prevalence, the more serious long-term emotional and physical consequences, and the greater potential for: A. fleeing to a shelter. B. homicide. C. possessive behavior. D. spontaneous abortion.

B. homicide.

ethic of care

Belief in the morality of responsibility in relationships that emphasize connection and caring

Cultural attitudes

Beliefs and perspectives that a society values.

The nurse must ask about violence at each prenatal and postpartum visit, especially with vulnerable populations such as teenagers, as well as observe for signs of violence on each visit. The nurse should be aware that the peak for postpartum intimate partner violence may be observed at various times according to ethnic group, such as: A. 6 months for white mothers. B. 10 months for white mothers. C. 3 months for African-American and Hispanic/Latino mothers. D. 12 months for African-American and Hispanic/Latino mothers.

C. 3 months for African-American and Hispanic/Latino mothers.

Within a state, counties designated as Health Professional Shortage Areas (HPSAs) tend to have a high proportion of racial minorities and fewer specialists. This factor may explain conflicting data within a state related to: A. adult immunization rates. B. chronic respiratory illness rates. C. maternal/infant morbidity rates. D. obesity rates.

C. Maternal/infant morbidity rates. HPSAs tend to have a high proportion of racial minorities and fewer specialists, such as pediatricians, obstetricians, and gynecologists, available to provide care to at-risk populations. There are extreme variations in pregnancy outcomes from one part of the country to another, and even within states.DIF: Cognitive Level: KnowledgeREF: Page 393

Intermittent or continuous resource

Cases may be exposed periodically or uninterrupted over a period of days or weeks.

Outbreak

Change (increase) in a disease and/or an event from expected levels to levels that are clearly in excess of expected levels.

General approaches to quality improvement

Seek to protect the public by ensuring a level of competency among health care professionals Examples: Credentialing, licesnsure, accreditation, certifications, charter, recognition, academic degrees.

sexual abuse

Coerced sexual acts ranging from fondling to rape or sexual degradation; it can happen to children or adults and be perpetrated by anyone inside or outside the family.

Rural

Communities having fewer than 20,000 residents or fewer than 99 persons per square mile.

Cross-tolerance

Condition in which tolerance to one drug results in a decreased response to another drug in the same general category.

The goal of deinstitutionalization was to improve the quality of life for people with mental disorders by providing services in the communities where they lived rather than in large institutions. At what program level did this change in locus of care fail? A. Assessment level B. Design level C. Evaluation level D. Implementation level

D. Implementation level

A nurse in community health working in an inner city clinic with high poverty and unemployment rates recognizes the need for programs for pregnant women because these women often receive late or no prenatal care and deliver: A. at home. B. full term. C. identical twins or triplets. D. low-birthweight babies.

D. low-birthweight babies.

A large industrial plant has recently laid off a significant portion of its workforce because of scalebacks in production. The occupational health nurse proposes education sessions with the remaining employees about effective strategies for managing stress during economic downturns. This suggestion to management is based on the nurse's understanding that increases in aggression and violence at home and work may be triggered by: A. competition. B. unemployment. C. survivor guilt. D. work-related stress.

D. work-related stress.

Federal Income poverty guidelines

Definition of poverty drafted by the Social Security Administration in 1964. The federal government defines poverty in terms of income, family size, the age of the head of household, and the number of children younger than 18 years. The guidelines change annually to be consistent with the consumer price index.

Basic program planning

Definition of problem, identify objectives, evaluate problem solutions and choose solution

According to the Minnesota Model of Public Health interventions, what is the first step of surveillance for the public health nurse?

Determine if surveillance is appropriate.

Endemic

Disease or an event that is found to be present (occurring) in a population in which there is a persistent (usual) presence with low to moderate disease or event cases; the constant presence of an infectious disease within a specific geographic area.

Hyperendemic

Disease or event that is found to have a persistently (usually) high number of cases.

Epidemiology and Surveillance of HIV/AIDS

Disproportionately affects minorities Worldwide 38 million persons live with HIV infection - 2/3s of those live in sub-Saharan Africa - About 12 million people in the U.S. --20% of those are unaware of their infection Heterosexuals account for 27% of estimated new cases (2009) More concentrated in urban areas

Program evaluation process

Engage stakeholders, describe program, focus the evaluation design, gather credible evidence, justify conclusions and ensure use and share lessons planned

Homeless persons

Federal government defines a homeless person as one who lacks a fixed, regular, and adequate address or has a primary nighttime residence in a supervised publicly or privately operated shelter for temporary accommodations.

federal public health agencies

Federal-level government agencies that develop regulations to implement policies formulated by Congress and provide a significant amount of funding to state and territorial health agencies for public health activities

Migrant Health Centers

Federally funded primary care centers to serve migrant populations.

Levels of program objectives

First level- general and broad Then specific objectives

The historical figure whose work in providing care to poor people, primary prevention, and environmental health established that individual as nursing's first moral leader and community-oriented nurse was:

Florence Nightingale.

elder abuse

Form of family violence against older members. It may include neglect and failure to provide adequate food, clothing, shelter, and physical and safety needs; it can also include roughness in care and actual violent behavior toward the elderly.

Urban

Geographic areas described as non rural and having a higher population density, more than 99 persons per square mile; cities with a population of at least 20,000 but fewer than 50,000.

Summative evaluation

Happens at the end; are the needs for which the program was designed being met?; are the problems it was designed to solve being solved?

primary caregivers

Health care professionals who are primarily responsible for providing for the health care needs of clients

assessor

Health professional who uses data in a systematic way to help identify needs, questions to be addressed, abilities, and available resources

Holoendemic

Highly prevalent problem found in a population; commonly acquired early in life. The prevalence of this problem decreases as age increases.

Pesticide Exposure

Health risk to farmworkers who work in fields that have been treated with pesticides. Residue from pesticides also enters farmworkers' homes and their food. Risks include mild psychological and behavioral deficits and acute severe poisoning that can result in death.

outreach workers

Health workers who make a special, focused effort to find people with specific health problems for the purpose of increasing their access to health services

Licensure

Is a contract between the profession and the state whereby the profession is granted control over who can enter into and who exists from the profession. Controls entry into a profession.

Outbreak Detection

Identifying a rise in the frequency of a disease above the usual occurrence of the disease.

Caring for AIDS Patients in the Community

Identifying resources such as social and financial support services Interpreting school and work policies Assisting employers by educating managers about how to deal with ill or infected workers to reduce the risk of breaching confidentiality or wrongful actions such as termination

A

In giving care to the survivors of violence, the nurse should demonstrate respect and caring for all family members, insist that safety is the first priority, and demonstrate intolerance for violent behavior. Additionally, the nurse should be: A. absolutely honest about what will be reported and what the family can expect. B. authoritarian in approaching the problem. C. cautious in reporting unconfirmed reports of violence. D. sincere in concern for the victims.

Syphilis

Infects moist mucous or cutaneous membranes Spread through direct contact, usually by sexual contact or from mother to fetus - Transmission via blood transfusion may occur if the donor is in the early stages of disease Highest rates are among MSM; recently, number of infected women has increased Clinical signs divided into primary, secondary, and tertiary infections Congenital syphilis

Alcoholics Anonymous (AA)

Lay, self-help group that practices a 12-step approach to recovery for persons with alcoholism.

communitarianism

Maintains that abstract, universal principles are not an adequate basis for moral decision making. History, tradition, and concrete moral communities should be the basis or moral thinking and action

ethical decision making

Making decisions within an orderly framework that considers contact, ethical approaches, client values, and professional obligations

Total quality Management

Management philosophy that includes a focus on client, continuous quality improvement (CQI) and teamwork.

Charter

Mechanism by which a state governmental agency grants corporate status to institutions with or without rights to award degrees.

Hepatitis A Virus

Most often transmitted through the fecal-oral route; sources may be water, food, or sexual contact; often silent in children

Relevance

Need for program

Gynecological age

Number of years from menarche

Program

Organized approach to meet the assessed needs of individuals to eliminate one or more problems

public health

Organized community efforts designed to prevent disease and promote health. It links disciplines, builds on the science of epidemiology, and focuses on the community; organized efforts designed to fulfill society's interest in ensuring conditions in which people can be healthy. It can be what members of society do collectively to ensure conditions that support health

Common Source Outbreak

Outbreak in which a group is exposed to a common noxious influence such as the release of noxious gases.

Propagated Outbreak

Outbreak that does not have a common source and spreads gradually from person to person over more than one incubation period.

Mixed Outbreak

Outbreak with a common source followed by secondary exposures related to person-to-person contact, as in the spreading of influenza.

The hospital infection control practitioner reports new cases of tuberculosis to the public health department. Of what type of surveillance system is this an example?

Passive

Level of Prevention-pneumocystis jiroveci pneumonia chemoprophylaxis for people with AIDS

Tertiary Prevention

Drug addiction

Pattern of abuse characterized by an overwhelming preoccupation with the use (compulsive use) of a drug and securing its supply and a high tendency for relapse if the drug is removed.

Sporadic

Problems with an irregular pattern with occasional cases found at irregular intervals.

Recognition

Process by which one agency accepts the credentialing status of and the credentials conferred by another agency.

Detoxification

Process of allowing time for the body to metabolize and/or excrete accumulations of a drug. It is often called social detoxification if the withdrawal symptoms are not life-threatening and do not require medication or medical detoxification if the symptoms require medical management.

Adequacy

Program addresses the extent of the need

MAPP goal

Program planning in public health arena to helps the community health workers be facilitators, as communities establish priorities in their public health issues and identify resources to address the issues

public health programs

Programs designed with the goal of improving a population's health status

A

Protecting the nation's food supply from contamination by all the virulent microbes is complex, costly, and time consuming. However, much foodborne illness, regardless of causal organisms, can be prevented through simple changes in: a. food preparation, handling, and storage. b. importation regulations. c. pesticide usage. d. animal breeding practices.

What are the purposes of disease surveillance in public health?

Provides a means for nurses to monitor disease trends. Generates knowledge about disease or outbreak patterns. Reduces morbidity and mortality and improves health through disease trend monitoring.

What is the purpose of using an algorithm in the surveillance process?

Provides the nurse with a step by step plan to identify events needing investigation.

distributive justice

Requires that there be a fair distribution of the benefits and burdens in society based on the needs and contributions of its members. This principle requires that consistent with the dignity and worth of its members and within the limits imposed by its resources, a society must determine a minimal level of goods and services to be available to its members

Epidemic

Rate of a disease within an area that is clearly in excess of expected levels (endemic) for a given period; the occurrence of an infectious agent or disease within a specific geographic area in greater numbers than would normally be expected.

Mental illness

Refers to all diagnosable mental disorders; it can affect persons of all ages, races, cultures, socioeconomic levels, and educational levels and persons of both genders.

Poverty

Refers to having insufficient financial resources to meet basic living expenses. These expenses include cost of food, shelter, clothing, transportation, and medical care.

Persistent poverty

Refers to individuals and families who remain poor for long periods

Neighborhood poverty

Refers to spatially defined areas of high poverty, characterized by dilapidated housing and high levels of unemployment.

Efficiency

Relationship between program outcomes and resources spent

Aspects of program evaluation

Relevance, adequacy, progress, efficiency, efficiency, effectiveness, impact and sustainability

Level of Prevention-sexually transmitted disease or infection or partner notification

Secondary Prevention

Level of Prevention-tuberculosis screening for health care workers

Secondary Prevention

advocacy

Set of actions undertaken on behalf of another while supporting the other's right to self-determination; activities for the purpose of protecting the rights of others while supporting the client's responsibility for self-determination; involves informing, supporting, and affirming a client's self-determination in health care decisions

incest

Sexual abuse among family members, typically a parent and a child.

rape

Sexual intercourse forced on an unwilling person by threat of bodily injury or loss of life.

morality

Shared and generational societal norms about what constitutes right or wrong conduct

Crisis poverty

Situation of hardship and struggle; it may be transient or episodic and can result from lack of employment, lack of education, domestic violence, or similar issues. These issues can lead to persistent poverty.

B D

Six students order meals at a local restaurant. Which of the following students are at highest risk for illness? (Select all that apply.) a. The first student asks for a salad with chicken strips and dressing on the side. b. The second student asks for a hamburger, very rare. c. The third student orders a tuna salad sandwich with extra mayonnaise. d. The fourth student orders a breakfast meal with two very soft-poached eggs and toast.

Syndronic Surveillance System

Systems developed to monitor illness syndromes or events, such as increased numbers of medication purchases, trips to physicians or emergency departments, or orders for cultures or radiographs, as well as rising levels of school or work absenteeism, which may indicate that an epidemic is developing hours or days before disease clusters are recognized or specific diagnoses are made and reported to public health agencies.

B D

The advanced practice nurse explains that the client has an upper respiratory infection (URI) and suggests several measures that might make the client more comfortable. Which of the following best describes why the nurse doesn't just prescribe antibiotics as the client repeatedly requests? (Select all that apply.) a. Antibiotics are expensive, whereas the support measures would be almost free of cost. b. Viral diseases are not affected by antibiotics. c. Clinics cannot afford to continually give antibiotics to anyone who asks for them. d. The more antibiotics are prescribed, the more infectious agents develop resistance to such drugs.

D

The goal of deinstitutionalization was to improve the quality of life for people with mental disorders by providing services in the communities where they lived rather than in large institutions. At what program level did this change in locus of care fail? A. Assessment level B. Design level C. Evaluation level D. Implementation level

B

The harm reduction approach to substance abuse focuses on health promotion and disease prevention. A primary prevention strategy that can be used by the nurse to address substance abuse under this approach is to: A. assess for recreational drug use. B. destroy the myth of good drugs versus bad drugs. C. encourage children to "just say no." D. refer to an addiction treatment program.

B

The nurse at the adult day care center notices bruises on the wrists of a 90-year-old client. Besides the physical assessment of the client, the nurse should: A. confront the daughter when she arrives to pick the father up. B. discuss the findings with the caregivers to determine the cause of the injuries. C. educate the staff about indications of elder abuse. D. make a referral to the primary care provider for follow-up.

Human Immunodeficiency Virus Infection (HIV)

The primary infections (within about 1 month of contracting the virus) Clinical latency (body shows no symptoms) A final stage of symptomatic disease

D

There is great concern in the nurse's community over three local cases of West Nile virus. Which of the following actions should the nurse take to get the community involved in addressing this problem? a. Ask the state department of health for assistance b. Demand that everyone over age 65 become immunized immediately c. Encourage immunization of all children under 12 d. Have an educational campaign to remove any containers of standing water

B

To reach the desired goal of maximizing the full immunization rates for preventable communicable disease and increasing herd immunity levels, it is crucial for the nurse to: A. assume that the primary care physician has provided all appropriate immunizations. B. check an individual's immunization status at each and every visit. C. support parents who are reluctant to immunize their children. D. understand the difficulties in obtaining and maintaining immunization schedules.

moral distress

Uncomfortable state of self when one is unable to act ethically

Substance Abuse

Use of any substance that threatens a person's health or impairs his or her social or economic functioning.

Community health report card

Useful tool to help identify areas where change is needed, to set priorities for actions, and to track changes in population over time. We can track leading causes of morbidity and mortality, looking at trends to see if public health interventions have improved health care outcomes.

C

Which of the following agencies are responsible for implementing and enforcing local, state, and federal public health codes and ordinances and providing essential public health programs to a community? A. Community health clinics B. Federal health agencies C. Local health agencies D. State health agencies

C

Within a state, counties designated as Health Professional Shortage Areas (HPSAs) tend to have a high proportion of racial minorities and fewer specialists. This factor may explain conflicting data within a state related to: A. adult immunization rates. B. chronic respiratory illness rates. C. maternal/infant morbidity rates. D. obesity rates.

feminists

Women and men who hold a worldview advocating economic, social and political status for women that is equivalent to that of men

Pandemic

Worldwide outbreak of an epidemic disease; refers to the epidemic spread of the problem over several countries or continents (e.g., the SARS outbreak).

HEDIS Health Plan Employer Data and information Set

a data-collection arm of the national committee for Quality Assurance (NCQA), providers performance information, or report cards, for managed care organizations (MCOs)

invasiveness

ability to penetrate and spread throughout a tissue

toxicity

ability to produce a poisonous reaction

The ethical question, "Are persons assigned to develop community knowledge adequately prepared to collect data on groups and populations?" is based on an ethical tenet that supports the community-oriented core function of:

assessment.

stereotyping

attributing certain beliefs and behaviors about a group to an individual w/o giving adequate attn to individual differences

A community-based hospice nurse has an Asian male client with terminal stage cancer. The client complains that he is in continuous pain and receives no relief from the codeine prescribed by his primary care practitioner (PCP). The nurse contacts the client's PCP to discuss replacing the prescription of codeine with another pain-reducing non-codeine-based drug. The nurse is demonstrating knowledge of the cultural organizing factor of:

biological variance.

The community health nurse performs an assessment of violence by observing which of the following community characteristics? Select all that apply. Presence of social support networks Crime rates Levels of unemployment Presence of physical disabilities in individuals Presence of family violence

crime rates levels of unemployment Identification of risk factors is an important part of primary prevention used by nurses who work with clients in a variety of settings. Although abuse cannot be predicted with certainty, several factors influence the onset and support the continuation of abusive patterns. Assessing for violence in a community context is completed by observing community characteristics inclusive of crime rates, unemployment levels, lack of neighborhood resource and support systems, and a lack of community cohesiveness. Individual factors include the presence of physical disability and familial factors are those related to violence and other family factors.DIF: Cognitive Level: ComprehensionREF: Page 470 (Box 25-3)

A goal of Healthy People 2020

decrease health disparities

The harm reduction approach to substance abuse focuses on health promotion and disease prevention. A primary prevention strategy that can be used by the nurse to address substance abuse under this approach is to: assess for recreational drug use. destroy the myth of good drugs versus bad drugs. encourage children to "just say no." refer to an addiction treatment program.

destroy the myth of good drugs versus bad drugs. Rationale: Nurses are experts in medication administration and understand the potential dangers of indiscriminate drug use and the inherent inability of drugs to cure problems. Nurses can influence the health of clients by destroying the "good drugs versus bad drugs"myth. This means (1) teaching clients that no drug is completely safe and that any drug can be abused and (2) helping persons learn how to make informed decisions about their drug use to minimize potential harm.DIF: Cognitive Level: ApplicationREF: Page 441

Caring and the ethic of care is a core value of nursing in community health. This ethical view was developed in the mid-1980s and early 1990s in response to the technical advances in health care science and the desire of nurses to:

differentiate nursing practice from medical practice.

A nurse overhears the doctor saying, "Let's not give him codeine; he is Asian." The nurse reflects on the comment and determines that the: (select all that apply)

doctor is culturally competent. doctor is planning care based on racial enzymatic differences.

endemic

efers to the constant presence of a disease within a geographic area or a population...from lecture—endemic is something that is naturally occurring in the population...pertussis is endemic in our population.

cultural skill

effective integration of cultural awareness and knowledge to obtain relevant cultural data and meet needs of diverse pts

prejudice

having a deeply held rxn, usually negative, about another group or person

Paternity

fatherhood

refugees

fear persecution in homeland

cultural shock

feeling of helplessness, discomfort, and disorientation experienced by an individual attempting to understand or adapt to another cultural group

Violence is a major public health problem in our communities that causes premature mortality and lifelong disability. Violence-related morbidity is a significant factor in: community deterioration. health care costs. juvenile delinquency. population density.

health care costs. Violence is a public health problem that has both emotional and physical effects. Violence is the major cause of premature mortality and lifelong disability, and violence-related morbidity is a significant factor in health care costs. Violent behavior is predictable, and therefore it is preventable, especially with community action.DIF: Cognitive Level: ApplicationREF: Pages 453-454

cultural imposition

imposing one's values on other; ignoring pt's values

Total Quality Management (TQM) and Continuous Quality Improvement (CQI)

is a management philosophy used in health care It is prevention oriented and process evaluation focused The goal of these two managements it to eliminate errors in the work process before negative outcomes occur rather than waiting until after the fact to correct individual performance; the focus is on problem prevention and continuous improvement.

Credentialing

is generally defined as the formal recognition of a person as a professional with technical competence or of an agency that has met minimum standards of performance.

The nurse in community health prepares a community assessment for the local planning commission and presents data that indicate an increase in violence among young children and teens within the community. The nurse projects the cost of care related to violence in ED visits, treatment of stabbing and gunshot wounds, and rehabilitation. The nurse also produces information on school absenteeism, graduation rates, and teenage rape and pregnancy. The planning commission creates a task force to explore the community outlets for child and teen recreation and the current community education programs available to families and students related to violence prevention, negotiation, and mediation. This can best be described as an example of:

power of partnerships.

The broad practice of public health requires a set of skills, knowledge, and attitudes in all disciplines. The baccalaureate level of entry for nursing is currently endorsed to achieve the core competencies of public health because of:

rapid change and limited resources in public health.

Goal of Tertiary Prevention related to infectious disease

reduce the complications and disabilities through treatment and rehabilitation--this may include helping people recover and return to their previous or a new level of health, as well as primary and secondary care to prevent the continuation of the infectious disease and its spread.

herd immunity

refers to the immunity of a group or community and is based on the resistance of a big proportion of individual members of a group to infection.

It is important that nurses understand the nontraditional healing practices of their clients because:

safe, effective nontraditional healing methods can be blended with Western medicine.

cultrural awareness

self-exam and in depth exploration of own beliefs/values as they influence behavior; understanding own culture

culture

set of beliefs, values, and assumptions about life and transmitted across generations; "learned worldview"; develops overtime and is resistant to change

race

social classification r/t physical markers

surveillance

the gathering of the who, what, when, and where, so that you can answer the why.

elimination

the goal is to remove a disease from a large geographic area such as a country or region of the world

communicable period

the interval during which an infectious agent may be transferred directly or indirectly from an infected person to another person. (Ex. the communicability of the flu is 3-5 days after the clinical onset of symptoms OR Hep B infected persons are infectious many weeks before the first symptoms and remain infective during the acute and chronic carrier state--maybe for life!)

eradication

the irreversible termination of all transmission of infection by extermination of the infectious agents worldwide.

Donabedian's Strucutre-process-outcome model the sentinel model and the tracer model

what are the three major models that have been used to evaluate quality

The community-oriented nursing care function of policy development is supported by the ethical tenet of:

what is ethical is also good policy.

A large industrial plant has recently laid off a significant portion of its workforce because of scalebacks in production. The occupational health nurse proposes education sessions with the remaining employees about effective strategies for managing stress during economic downturns. This suggestion to management is based on the nurse's understanding that increases in aggression and violence at home and work may be triggered by: competition. unemployment. survivor guilt. work-related stress.

work-related stress. Productive and paid work is an expectation in mainstream American society. Work can be fulfilling and contribute to a sense of well-being; it can also be frustrating and unfulfilling, contributing to stress that may lead to aggression and violence. Some people are frustrated by jobs that are repetitive, boring, and lack stimulation.DIF: Cognitive Level: ApplicationREF: Pages 454-455

A nurse demonstrates cultural competence by using statements such as:

"Tell me about your health care beliefs."

ten basic elements of surveillance

1. mortality registration 2. morbidity reporting 3. epidemic reporting 4. epidemic field investigation 5. laboratory reporting 6. individual case investigation 7. surveys 8. usage of biological agents and drugs 9. distribution of animal reservoirs and vectors 10. demographic and environmental data

standards for culturally competent practice

1. social justice 2.critical reflection 3.knowledge of cultures 4. culturally competent practice 5. cultural competence in health care system 6. advocacy and empowerment 7. multicultural workforce 8. edu & training in competent care 9. cross-cultural communication 10. cross-cultural leadership 11. policy development 12. EBP and research

13. With land priced much lower in rural areas than in urban areas, why don't companies build work places in rural areas where people are anxious to find full-time employment? a. Employers need highly educated and skilled employees. b. Managers don't want to be transferred into a rural area where they won't fit in and won't be comfortable. c. Rural persons don't work as hard as urban dwellers. d. Smells emanating from farms are unacceptable to persons working inside.

A

16. As a concerned citizen and professional nurse, you will be encouraging your senators and representatives to support expansion of health clinics for the poverty stricken. Where is the most desperate need at this point in time? a. Rural areas where migrant and seasonal farm workers labor b. Inner-city poverty-stricken neighborhoods c. Inside elderly low-income housing units d. Inside high schools in poverty-stricken neighborhoods

A

21. A nurse was becoming very frustrated at the ongoing stream of farmworkers with respiratory problems each spring. What might the nurse consider doing? a. Community education on herbicides and other chemicals spread each spring with information on self-protective measures such as respirators b. Fight to get farmers to quit spreading chemicals on their fields each spring. c. Suggest that farmers spray water on the fields immediately after the chemicals are spread to reduce dust in the air. d. Tell farm workers to obtain different employment. e. Try to get farmers to reduce the amount of chemicals used on their fields.

A

22. A nurse was trying to decrease the amount of respiratory illness in the community. Which approach should the nurse use? a. Attend each farmworkers' meeting and listen to how they perceive and discuss their problems b. Offer to teach the farmers' spouses how to protect their families, such as having the farmer shower and change clothes before coming into the house c. Suggest the community work together to obtain a community college so farmworkers could be educated for other employment opportunities d. Teach children in the school system the importance of seeking employment in cities rather than staying on the farm

A

26. Looking at health care throughout the United States, approximately what proportion of health care funds is dedicated to public health services? a. Less than 5% b. Between 6% and 10% c. Between 11% and 15% d. Between 16% and 20% e. Over 21%

A

3. In addition to those barriers faced by many residents in rural areas, what additional barrier to health care is a Hispanic migrant farmworker likely to encounter? a. Absence of culturally competent care b. Availability of specialists c. Distance of health care facilities from the place of residence d. High cost of health care

A

4. From whom is a rural resident with asthma most likely to receive health care services? a. Advanced nurse practitioner b. Allergist c. Pediatrician d. Pulmonologist

A

5. What is the practical importance of the new classification system of metropolitan, micropolitan, and noncare areas? a. Classifications are used for congressional policy-making and funding decisions. b. Metropolitan areas gained importance because of their large population. c. Micropolitan areas are now recognized for their crucial role in society. d. The new system emphasizes the extent of rural (noncore) areas.

A

6. Which compositional factor influences rural health needs? a. Demographics of the people living in an area b. Local environment c. The geography of the area d. The political and social institutions in the community

A

7. A Mexican immigrant mother tells the nurse she is concerned that people in the community will give her daughter mal de ojo. To what folk illness is he mother referring? a. Evil eye b. Fallen fontanel c. Soul loss d. Indigestion

A

8. What assistance might be available to a migrant farm family if the father becomes injured? a. Emergency department for immediate care b. Migrant Health Act clinics c. OSHA, since the man was injured by farm machinery d. Workers' Compensation, since injury was at work

A

A

A Mexican immigrant mother tells the nurse she is concerned that people in the community will give her daughter mal de ojo. To which of the following folk illnesses is the nurse referring? a. Evil eye b. Fallen fontanel c. Soul loss d. Indigestion

A

A case management nurse for a locally funded program for special-needs children is increasingly concerned about a recent referral for a 1-year-old child with a congenital illness residing in a poverty-stricken community. The nurse knows that this child may be at higher risk for the most harmful effects of poverty, including: A. developmental delays. B. ear infections. C. frequent colds and infections. D. irritability.

A

A client is back for his follow-up appointment and says to the nurse, "I know. I know. I drink too much, but the job is so stressful that I need to find a way to unwind at the end of the day. You would too!" The nurse should recognize that this is a primary symptom of addiction known as: A. denial. B. social drug use patterns. C. setting variable. D. coping.

A

A client is using a primary prevention strategy to prevent infectious disease. Which of the following actions is the client most likely taking? a. A client receives a tetanus booster every 10 years. b. A client receives a tetanus booster after stepping on a nail. c. A client receives tetanus immunoglobulin after stepping on a nail. d. A client with tetanus is given antibiotics and is placed on seizure precautions.

A

A client requests help to stop smoking. Which of the following methods would be the best for the nurse to suggest to the client? a. A combination of interventions, beginning with changing the environment b. Acupuncture to reduce the nicotine cravings c. Behavior modification through hypnosis d. Nicotine replacement products

D

A community health nurse is trying to decrease the incidence of emerging infections caused by drug-resistant organisms. Which of the following actions would be most appropriate for the nurse to implement? A. Lobby for increased use of antibiotics in the treatment and prevention of communicable diseases. B. Notify the Centers for Disease Control regarding any unusual signs and symptoms related to a communicable illness. C. Reinforce the importance of receiving vaccinations according to recommended schedules. D. Teach parents that antibiotics should not be used for treatment of viral infections or for prevention of bacterial infections.

C

A community is experiencing an epidemic of the measles. The nurse is trying to determine if this problem is happening in other communities as well. Which of the following resources should the nurse use to answer this question? a. Centers for Disease Control and Prevention Weekly Report b. Communicable Diseases Weekly Report c. Morbidity and Mortality Weekly Report d. Weekly National Report of Communicable Diseases

C

A community mental health nurse is implementing a tertiary prevention activity. Which of the following interventions is most likely being completed? A. Disseminating information about mental health to community organizations B. Partnering with school health nurses for early identification of children who have evidence of mental health problems C. Providing case management services that link clients with serious mental illnesses to mental health and community support services D. Screening high-risk clients for the presence of mental disorders

C

A community mental health nurse is working within the community to help prevent youth violence. Which of the following interventions would most likely be implemented by the nurse? a. Distributing literature that associates violence with a lack of intelligence b. Explaining to youth why youth violence is detrimental to society c. Partnering with associations to provide alternative activities that improve social skills d. Recommending increased funding to prosecute and jail teens associated with violent activity

C

A father brings his stepdaughter to the family clinic for an immunization update before the new school year. The nurse notices the interaction between the young girl and her stepfather. The child appears tense and cautious and wraps her arms around herself in a protective manner. The child startles when touched by the stepfather and pulls away. The stepfather is overheard saying, "I'll leave you here if you don't behave and act nice." In this scenario, it would be important for the nurse to explore in her assessment the possibility of: A. child neglect. B. family secrets. C. father-daughter incest. D. impaired family functioning.

B

A home health client has been taking 50 mg of meperidine, a narcotic analgesic, every 4 hours for pain. Which of the following statements made by a nurse to the client's physician indicates a need for further education on narcotics? a. "You need to decrease the time interval between doses to every 3 hours so that my client will maintain pain relief." b. "You need to discontinue that order before my client becomes addicted." c. "You need to increase the meperidine so that my client will achieve adequate pain relief." d. "You need to give my client something in addition to the meperidine."

D

A home health nurse believes a client may be a drug addict. However, the client denies the use of drugs when questioned by the nurse. Which of the following is the most likely cause of the client's denial? a. Confusion resulting from dementia b. History of compulsive lying c. Mental status changes d. Use of addictive drugs

B

A hospice nurse is working with a cancer client and his family. The client's 7-year-old son has developed recent school and peer problems. Understanding the risk for disruption of normal development, the nurse plans to first screen the child for: A. developmental disorders. B. mental health problems. C. parental neglect. D. violence potential.

D

A large industrial plant has recently laid off a significant portion of its workforce because of scalebacks in production. The occupational health nurse proposes education sessions with the remaining employees about effective strategies for managing stress during economic downturns. This suggestion to management is based on the nurse's understanding that increases in aggression and violence at home and work may be triggered by: A. competition. B. unemployment. C. survivor guilt. D. work-related stress.

C

A long-distance truck driver being assessed by a nurse in a community-based clinic states, "I smoke 3 packs of cigarettes a day. I use coffee and diet pills from the drug store to stay awake on the road. That makes it difficult to sleep when I do pull over, so I use a prescription sleeping pill from my doctor to sleep for 4 hours. It's giving me palpitations." The nurse's assessment should include a diagnosis of: A. drug abuse. B. drug addiction. C. substance abuse. D. adverse drug reaction.

d

A major emerging public health issue facing nursing in community health is: a. bioterrorism. b. health literacy. c. health ministries. d. welfare reform.

D

A male IV drug abuser admits to the nurse that he has no desire to stop using drugs, so rather than lecture him on the dangers of drug addiction, the nurse counsels him on how to sterilize his needles. Which of the following prevention efforts is the nurse attempting to achieve? a. Primary prevention by educating about safe injections b. Primary prevention by avoidance of future legal complications c. Secondary prevention to reduce the risk for infection or other complications d. Tertiary prevention to reduce the transmission of blood-borne diseases

D

A man is addicted to alcohol. In which of the following scenarios is a family member enabling the man to continue drinking? a. The wife asks the nurse to explain why her husband's continued drinking is dangerous. b. The son threatens to leave home because he finds the father's behavior embarrassing. c. The teenage daughter turns to a favorite teacher for emotional support. d. The wife tells her husband's boss that her husband is sick when he is actually inebriated.

C

A man loudly protests his increased property tax bill right after the public health department has made a plea for more funds. "Why," he asks, "should my tax dollars be used to pay for their children to be immunized?" Which of the following would be the best response by the nurse? a. "Immunizations are required by law, and if their parents can't afford it, you and I will have to pay for it." b. "It's just the right thing to do." c. "Only by making sure most kids are immunized can we stop epidemics that might hurt all of us." d. "We're a religious God-fearing community, and we take care of each other."

C

A man who takes a prescribed narcotic for pain on a regular basis reports that he does not like the way the medicine makes him feel. He has tried to change to an alternate analgesic but experienced withdrawal symptoms when he stopped taking the narcotic. Which of the following problems is the man experiencing? a. Drug abuse b. Drug addiction c. Drug dependence d. Substance abuse

C

A migrant farmworker brings his daughter into the clinic with severe heat stroke from being out in the sun. The nurse explains the danger signs and stresses staying cool and drinking lots of water. The man seems to know this already. Which of the following best describes the most likely reason that this happened? a. It was a rare occurrence, which probably won't be repeated. b. The daughter either disobeyed her father and went out to play in the sun or just did not realize how hot she was. c. Children may work on small farms because the family may need the additional income. d. The parents were busy working and didn't realize the child was outside so long.

A

A migrant farmworker has been injured while working on the farm. Which of the following would provide assistance to care for this injury? a. Emergency department for immediate care b. Migrant Health Act clinics c. OSHA, because the man was injured by farm machinery d. Workers' Compensation, because injury was at work

C

A migrant farmworker presents to the clinic reporting an acute onset of severe abdominal pain, nausea, vomiting, diarrhea, and headache with difficulty concentrating. Which of the following conditions would cause such symptoms? a. Appendicitis b. Bacterial gastroenteritis c. Pesticide poisoning d. Viral illness

D

A mother and her son are in the emergency department. The mother is extremely upset. Earlier, the son had become so angry that he put his hand through a glass window and had to have stitches. The physician suggested a psychiatric consult. Now the mother asks, "Do you think my son is crazy?" Which of the following is the most appropriate response by the nurse? a. "Absolutely not. But a psychiatrist can help your son realize more appropriate ways of displaying anger." b. "Have you had other reasons to think your son is crazy? Perhaps the psychiatrist can reassure you that your son is just an adolescent coping with hormones." c. "Your son is having problems coping with anger. And naturally you're upset. Let the psychiatrist determine whether your son is crazy or not." d. "Your son is having problems with behavior, which is distressing all of you. A psychiatrist may be able to help your son cope with life in a more acceptable way."

D

A mother cannot understand why her daughter ran away from home. When the police locate the daughter, she complains of a severe headache, so the police take both of them to the emergency department for assessment. Which of the following potential problems should be assessed by the nurse? a. The daughter's inability to do well in school b. Intimacy problems with the daughter's boyfriend c. The mother's abuse of the daughter over a long period of time d. Sexual abuse by the father

D

A mother confides to the nurse that her live-in boyfriend knocked down her 2-year-old child because he was crying too much. She begs the nurse not to tell anyone because her boyfriend has agreed to take anger management classes. Which of the following actions should be taken by the nurse? a. Abide by the mother's wishes because this information was provided in confidence. b. Arrange for counseling for the boyfriend at the earliest possible availability. c. Encourage the mother to take the child and find alternate housing right away. d. Report the incident to child protective services or other appropriate legal authority.

B

A nurse in community health conducting a home visit notices a 4-year-old girl sitting on a stool in an adjoining room. The girl is quiet and withdrawn, rarely makes eye contact, and does not leave the room. The nurse proceeds to ask about the child and attempts to engage the child in conversation. The nurse is assessing for what indicators of child abuse? A. Emotional abuse B. Emotional neglect C. Physical abuse D. Physical neglect

D

A nurse in community health conducts quarterly mental health-promotion and depression-screening programs at the local senior center. The nurse is aware that older adults are at increased risk for developing depression. Using such an intervention also addresses the older adults': A. dependence on their primary care provider. B. normal sensory losses. C. reduced social contacts. D. underutilization of the mental health system.

B

A nurse in community health is following an older woman who complains frequently of migraine headaches, backaches, and GI disturbances. During a recent visit, the woman states that her adult son now lives with her. He was recently let go from his job. He enjoys hanging out with his old buddies. When he feels better he will look for a new job. Despite the added burden she really enjoys having her son around again. The nurse should explore this further to determine whether the family is: A. abusive. B. codependent. C. coping. D. estranged.

C

A nurse in community health is working in a rural setting. In planning for programs to address the population's needs, the nurse should be aware that, in general, rural populations: A. engage in physical activity during leisure time. B. engage in preventive behaviors. C. perceive their overall health as less favorable. D. use seat belts.

A

A nurse in community health is working with a parent whose spouse has been called up for active duty in the military reserve. The family is experiencing financial strain due to decreased income. The extended family lives at a distance. The parent is struggling to manage the family in the spouse's absence. The family consists of four children (three preschool and one preteen). In this situation, it would be important for the nurse to further explore the potential for: A. child abuse. B. depression. C. intimate partner abuse. D. parent's resentment of the preteen.

A

A nurse in community health located in Virginia is conducting an assessment on a Hispanic worker currently working in a local apple orchard for the season. The nurse determines that the worker originates from Florida and is living in temporary housing with other orchard workers. Based on this information, the nurse should integrate the special needs of what vulnerable population? A. Migrant farmworkers B. Seasonal farmworkers C. Underinsured D. Undocumented aliens

D

A nurse in community health working in an inner city clinic with high poverty and unemployment rates recognizes the need for programs for pregnant women because these women often receive late or no prenatal care and deliver: A. at home. B. full term. C. identical twins or triplets. D. low-birthweight babies.

C

A nurse is assessing a young, single pregnant woman. Which of the following findings would be of most concern? a. Elevated blood pressure b. First prenatal visit at 5 months' gestation c. Persistent homelessness d. Positive STD test

BC

A nurse is caring for Mexican migrant farmworkers. Which of the following conditions are of greatest importance for the nurse to assess? (Select all that apply.) a. Cholera b. Hepatitis c. High blood level of lead d. Malaria

A

A nurse is caring for a migrant farm worker who has been working in the agricultural industry for the past 10 years. When questioned about environmental hazards, the client reports regular exposure to pesticides. Which of the following disorders is the client most at risk to develop? a. Cancer b. Memory loss c. Skin rashes d. Headaches

BD

A nurse is concerned about a teenage mother who does not seem to know how to play with an infant. Which of the following behaviors would the nurse want to role-model as good parenting behaviors? (Select all that apply.) a. Allow the baby to cry for 10 or 15 minutes before reacting so that the infant can learn to self-soothe. b. Cuddle and hold the baby while smiling and gazing into the baby's eyes. c. Teach the mother to keep her face about 4 or 5 inches from the baby and to gently blow on the infant's face. d. Sing to baby with different melodies until baby seems to listen and shows a preference for certain tunes.

A

A nurse is concerned about stress related to the heavy caregiving burden assumed by adult children of older clients. Which of the following actions by the nurse demonstrates the use of secondary prevention to limit caregiver stress? a. Asking caregivers how they are doing and suggesting coping strategies b. Encouraging caregivers to periodically leave the house for a couple of hours c. Establishing support groups for caregivers of older parents d. Referring some activities to neighbors and friends

C

A nurse is concerned about the possibility of suicide in a teenage boy. Which of the following characteristics has the boy most likely displayed? a. He has threatened harm to his school peers. b. He is between the ages of 15 and 19. c. He is depressed and has a history of being sexually abused. d. He has been accused of abusing an animal.

D

A nurse is employed at the state department of public health. Which of the following is a fundamental principle guiding his practice? A. Decisions are made at the federal level, disseminated at the state level, and carried out at the community level. B. Health of populations is only as good as the health of individuals that live and work in the community. C. Oversight of community health departments is the most important function. D. The interaction of the local-state-federal partnership is critical to success.

ABD

A nurse is employed by a state public health department. Which of the following functions would most likely be completed by this agency (Select all that apply.) A. Delegating power to engage in certain activities such as quarantine B. Enforcing laws and regulations that protect public health C. Filing birth and death certificates whenever these occur within the state D. Monitoring health status of citizens within its geographic boundaries

B

A nurse is implementing a primary prevention activity to decrease the incidence of communicable disease. Which of the following actions is the nurse most likely taking? A. Identifying and treating clients in a sexually transmitted disease clinic B. Partnering with schoolteachers to teach and observe hand washing techniques in elementary school children C. Providing case management services that link clients with communicable diseases to health care and community support services D. Performing directly observed therapy (DOT) for clients with active tuberculosis

D

A nurse is implementing a tertiary prevention strategy related to pesticide exposure. Which of the following activities would the nurse complete? a. Observe farmworkers for evidence of unsafe handling of pesticides. b. Provide teaching on how to handle pesticides to avoid or decrease exposure. c. Teach farmworkers how to recognize signs and symptoms of pesticide poisoning. d. Treat a client who has pesticide exposure to prevent complications.

A

A nurse is providing care to a pregnant teenager. Which of the following principles is the most important for the nurse to consider when providing care? a. All teen pregnancies are considered high-risk. b. Limited self-care knowledge can lead to pregnancy complications. c. Pregnant teens are likely to receive prenatal care late in the pregnancy. d. Pregnant teens may have limited financial resources to pay for care.

D

A nurse is reviewing with a client the importance of the diabetic diet to maintaining control of blood glucose levels. The client states, "I went to the classes and learned how to count carbohydrates. I have even started to write down what I eat for each meal." Which of the following would be most appropriate response from the nurse? a. "I'm so glad you're beginning to recognize the need to control your blood sugar." b. "Now you just have to make sure you don't give that habit up!" c. "Sounds like you're not ready to really plan your eating yet." d. "That is great. You've started to take action and make changes."

AD

A nurse is screening a client for ATOD problems. Which of the following actions would the nurse most likely complete? (Select all that apply.) a. Assist with identifying help or resources. b. Advise about the need to enter a treatment program. c. Ask about how relationships with family members and friends have been affected. d. Assess amount and pattern of use.

B

A nurse is working at a local health department. Which of the following would most likely be completed by this agency? A. Enforcement of laws and regulations related to public health nationwide B. Filing of a certificate for any birth or death in a community C. Providing experts in various specialties to answer questions and respond as needed D. Surveying the state's public health needs

B

A nurse is working with Mexican immigrants. Which of the following behaviors would most likely lead to a positive interaction for the nurse? a. Avoiding touching the client except when necessary as part of the physical examination b. Calling the client by name, socializing before addressing the problem, and being very respectful c. Keeping all interactions direct, to the point, and targeted on the reason for presentation d. Maintaining a non-confrontational relationship by avoiding any disagreement even if the nurse does disagree with what the client is saying

D

A nurse is working with a family to reduce the incidence of home violence. Which of the following actions by the nurse would most likely have a positive effect? a. Being assertive and taking control of decisions because family members cannot be expected to have this ability b. Focusing attention on family weaknesses that need to be fixed before healing can begin c. Pointing out things that victims should do differently to avoid antagonizing the abuser d. Soliciting ideas from the family regarding what they feel will improve their situation

C

A nurse new to the community evaluates the resources available to a father that has sought help with his escalating abuse and threats of violence to his family. After making the referral, the nurse approaches the local newspaper about running a series on the nature and extent of human abuse in the community. This strategy would: A. advocate for government programs to treat survivors. B. demonstrate the provider's commitment to address the need for services. C. increase awareness of community resources to address violence and abuse. D. increase the number of individuals identified as perpetrators.

B

A nurse sees the father of a newborn baby push his wife in a wheelchair out onto the patio. The man and woman each smoke a cigarette while the baby sleeps in the mother's arms. Which of the following information should the nurse share with the parents? a. Fetal nicotine syndrome may cause complications in the newborn. b. The sidestream smoke that the baby is exposed to is more dangerous than the mainstream smoke that the parents are inhaling themselves. c. The baby will eventually develop tolerance to tobacco smoke. d. The parents should switch to chewing tobacco as long as the child lives in the home.

ACD

A nurse tells her nursing supervisor that her family is moving from the urban area where they both live to a rural area to be near her spouse's parents, who are becoming less independent. Which of the following suggestions would the nursing supervisor provide to the nurse? (Select all that apply.) a. "Community members will probably hold you in higher regard and will look up to you." b. "Expect to have less autonomy in a small town than you have working as a nurse in a medical center." c. "You may feel like an isolated outsider, because the community may not immediately accept you." d. "You will have to be very sensitive about the differences in the rural lifestyle."

B

A nurse was shocked when late one night she saw a former high school friend going through a trash bin outside a fast-food restaurant and pulling out half-eaten food. Which of the following is the most likely explanation for a healthy young adult engaging in such behavior? a. He ate his meal but wanted more and had no more money. b. He had full-time employment, but with such a low salary it was inadequate to meet basic expenses. c. A friend had thrown his meal away, and he thought he could find his friend's untouched food. d. He was doing this to fulfill an obligation of fraternity initiation at the college.

ABC

A nurse's family has moved to a rural area to be near her spouse's aging parents. Which of the following would the nurse most likely notice about the family's new environment? (Select all that apply.) a. It is very difficult to obtain continuing education, except for online programs. b. Neighbors seem to expect the nurse to know everything about all health care and illness conditions. c. Neighbors ask questions about health concerns any time they see the nurse, regardless of where they are. d. People come to the health clinic before their situation becomes relatively serious.

C

A public health nurse is hosting an informational meeting for young parents about how to balance their multiple responsibilities. Which of the following parents in attendance would be most at risk for abusing his or her own child? a. A new mother who has joined a parents' support group b. A father who is overwhelmed and exhausted by overtime work c. A depressed father who was laid off from work and is missing his work colleagues d. A mother who is missing her career challenges and friends but enjoying being home with her baby

B

A public health nurse traces sexual contacts of clients with sexually transmitted diseases for screening and treatment. Which of the following levels of prevention is being implemented? A. Primary prevention B. Secondary prevention C. Tertiary prevention D. Secondary and tertiary prevention

A

A recent movie release portrays a criminal as a black female drug user whose abusive boyfriend has two children by different women. She lives in the riot-torn inner city of a large metropolitan area. This best demonstrates what type of factor that influences poverty? A. Cultural B. Environmental C. Political D. Social

B

A school health nurse is conducting drug screening to detect drug use by students before problems develop. Which of the following levels of prevention does this action represent? a. Primary prevention b. Secondary prevention c. Tertiary prevention d. Primary and secondary prevention

B

A school health nurse presents a program on preventing teen pregnancy to a group of parents. Following the presentation, which of the following comments by a parent would cause concern? a. "I do not know if my son is sexually active; however, I have decided I'm going to talk to him about it." b. "My daughter is too intelligent to get involved with boys, even if her friends do sleep around." c. "My daughter and I have often discussed sexuality, and when she's ready, I'll pay for her birth control pills. d. "I have spoken to my son about birth control. He says he's not ready to be a father and support a baby. He wants to go to college."

C

A school nurse is asked to assess a 13-year-old child because of excessive drowsiness and inattention in class. The nurse determines that this is a child of migrant workers on a local produce farm. Based on the nurse's knowledge of migrant worker families, the nurse should first explore the child's potential involvement in: A. delinquent behavior. B. drug use. C. field work. D. Migrant Education Program.

D

A school nurse stops a student in the hall and asks whether the student has been having trouble with her eyes, which look rather red and dry. The female student yawns and says that she hasn't noticed any eye problems. However, she does confess to the school nurse that she has gained 15 pounds since she began smoking "special cigarettes." Which of the following is the student most likely smoking? a. Tobacco b. Crack cocaine c. Ice methamphetamine d. Marijuana

B

A school nurse was talking to the teacher of an 8-year-old child who was living with her mother in their car. Which of the following concerns would lead the nurse to talk to the teacher about the child? a. Concern that the child is being mistreated by other children b. Concern that the child has developmental delays c. Concern that the child is given adequate food during lunch d. Concern that the child may need to sit in the front in order to be able to see well

B

A student comes to the college health clinic with typical cold symptoms of fever, sneezing, and coughing, but the nurse also notes small white spots on the inside of the student's cheeks. Which of the following actions should be taken by the college health nurse? a. Inform all students, staff, and faculty of a possible rubella epidemic b. Inform all students, staff, and faculty of a possible measles epidemic c. Reassure the student that it is just a bad cold and will soon pass d. Tell the student to take two acetaminophen and drink lots of fluids

B

A student complains to the college health nurse that her academic work has been going downhill because of lack of sleep. "My 3-year-old probably misses her babysitter since she has started going to the big daycare center. She hasn't been sleeping well and keeps scratching her bottom. Hopefully, she'll adapt to daycare soon." Which of the following information should the nurse provide to the student? a. "Dry skin in winter weather can cause itchiness; try to put on lotion before bedtime." b. "Your daughter may have pinworms; let me teach you how to check for this." c. "Perhaps your child is not developmentally ready for group play." d. "Try to arrange more one-on-one time with your 3-year-old."

B

A teacher sends a student to the school nurse because the student does not seem to be thinking clearly. The nurse notes that the student is having palpitations, as well as elevated pulse, blood pressure, temperature. Which of the following would most likely cause these symptoms? a. Alcohol b. Ecstasy (MDMA) c. Heroin d. Marijuana

CD

A teenager who has just come in for her prenatal visit appears to be about 6 months pregnant. Which of the following best describe why the teenage girl has waited so long to come in for prenatal care? (Select all that apply.) a. Her friends were suggesting ways to make the problem go away. b. She knew she could not be pregnant because her boyfriend said he used a condom. c. She was afraid her parents would pressure her to terminate the pregnancy. d. She kept hoping the pregnancy would just go away.

C

A woman has just delivered a baby and is lamenting that the baby's father is not with her. She shares with you that he became involved with drugs and is now in prison for theft. The new mother says, "It's so sad. He's clearly no good." Which of the following would be the best response by the nurse? a. "I'm so sorry to hear that. Is your mother able to be with you?" b. "It's good that you and the baby are safe from him." c. "Is there anyone to help you with the baby until the baby's father can be released and encouraged to get treatment for his addiction?" d. "You must hate him for leaving you alone with a new baby!"

34. What do expert rural nurses report as benefits of their work? Select all that apply. a. Ability to develop close relationships and give holistic care b. Being able to practice in the community where one lives c. Constant challenges d. Continual growth as an expert generalist e. Higher salary—although no additional benefits—than in urban areas f. Intensity of purpose

A, B, C, D, F

31. Why haven't for-profit managed care organizations begun to move into rural areas to increase the number of persons served? Select all that apply. a. High start-up and administrative costs are needed. b. Large number of elderly are on Medicare or Medicaid, which results in low reimbursement. c. Managed care is coming but hasn't yet had time to go beyond major population centers. d. Managed care organizations are waiting until federal reimbursements for underserved rural health areas become higher. e. Population is too small and too spread out. f. Rural health areas are too geographically unattractive and lack any recreational possibilities for managers to want to work there.

A, B, E

33. Why might there be more long-term consequences (such as death) to a driver injured in a truck accident in a rural area than a driver injured in a truck accident in an urban area? Select all that apply. a. Rural drivers are more likely to speed over long distances than urban drivers. b. Rural drivers aren't as skilled with vehicles as urban drivers, so they are more severely injured in a crash. c. The urban driver is taken immediately for care, whereas the rural driver might be injured on an isolated field and not found by anyone for hours. d. Urban drivers are more likely to use their seat belts than rural drivers. e. Urban drivers are probably rushed to an emergency department, whereas the rural driver lives far away from emergency services, often over bad roads and possibly through severe weather. f. Urban drivers trade their vehicles in rather frequently whereas rural drivers keep their vehicles for as long as they can make them run so the vehicles are much less roadworthy.

A, C, D, E

30. In comparison with mental health care in urban areas, how does mental health care in rural areas differ? Select all that apply. a. Access to care is more limited or not available at all locally. b. Cost for treatment is typically reimbursed at a lower rate. c. Depression is frequently not recognized or is not treated. d. Ethnic and racial disparities exist. e. Primary health care professionals accept responsibility for mental health treatment. f. Suicide rate is notably higher.

A, C, E, F

32. A nurse was excited about being the only staff member in the public health department in a small rural area. What problems might the nurse encounter? Select all that apply. a. No one with whom to collaborate or exchange ideas b. No physician available to tell the nurse what needed to be done c. No feedback can be obtained. d. The nurse may only be able to offer services supported by federal funds. e. The nurse will have to raise local funds to support the department. f. There won't be any epidemiologists or sanitarians to help.

A, D, F

17. A nurse's family has moved to a rural area to be near her spouse's aging parents. What might the nurse notice about the family's new environment? Select all that apply. a. It is very difficult to obtain continuing education, except for online programs. b. Neighbors seem to expect the nurse to know everything about all health care and illness conditions. c. Neighbors ask questions about health concerns any time they see the nurse, regardless of where they are. d. People come to the health clinic before their situation becomes relatively serious. e. People seem to feel perfectly comfortable asking the nurse for confidential information about other people who have been in to the clinic. f. The nurse feels that people simply do not appreciate her expertise and advice.

A,B,C,E

16. A friend who is a nurse tells you that her family is moving from the urban area where you both live to a rural area to be near her spouse's parents, who are becoming less independent. What advice would you give for the nurse in preparing for any possible nursing role changes? Select all that apply. a. "Community members will probably hold you in higher regard and will look up to you." b. "Expect to have less autonomy in a small town than you have working as a nurse in a medical center." c. "You may feel like an isolated outsider, since the community may not immediately accept you." d. "You will have to be very sensitive about the differences in the rural lifestyle." e. "You will have a surplus of resources and supplies because so few clients come in for services." f. "You will need to focus on developing your specialized knowledge and skills."

A,C,D

14. Other than being concerned for all U.S. citizens, why would health professionals be particularly concerned about the health needs of residents in rural areas? Select all that apply. a. About 25% of all U.S. residents live in rural settings. d. A high prevalence of poverty exists among rural families. f. Rural families are less likely to engage in health promotion and disease prevention activities.

A,D,F

A nurse in community health located in Virginia is conducting an assessment on a Hispanic worker currently working in a local apple orchard for the season. The nurse determines that the worker originates from Florida and is living in temporary housing with other orchard workers. Based on this information, the nurse should integrate the special needs of what vulnerable population? A. Migrant farmworkers B. Seasonal farmworkers C. Underinsured D. Undocumented aliens

A. Migrant farmworkers A migrant farmworker is a person whose principal employment is in agriculture on a seasonal basis, who has been employed within the last 24 months, and who establishes for the purpose of such employment a temporary abode. Seasonal farmworkers work cyclically in agriculture but do not migrate. The majority of migrant and seasonal farmworkers are foreign born and predominantly Mexican (75%), coming primarily from the west central states and more recently the states of southern Mexico.DIF: Cognitive Level: KnowledgeREF: Page 396

A case management nurse for a locally funded program for special-needs children is increasingly concerned about a recent referral for a 1-year-old child with a congenital illness residing in a poverty-stricken community. The nurse knows that this child may be at higher risk for the most harmful effects of poverty, including: A. developmental delays. B. ear infections. C. frequent colds and infections. D. irritability.

A. developmental delays.

For the homeless, health care is usually crisis oriented and sought in emergency departments. The most difficult challenge for nurses treating this vulnerable population is to recognize the client's: A. limitations in following treatment protocols. B. limited number of transient treatment facilities. C. transition to persistent poverty. D. use and abuse of tobacco, alcohol, and illicit drugs.

A. limitations in following treatment protocols.

When determining whether a geographic area is rural or urban, the nurse should recognize that: A. rural and urban areas, by relative nature, occur on a continuum. B. rural regions have fewer than six persons per square mile. C. rural residents feel isolated. D. rural areas are recreational, retirement, or resort communities.

A. rural and urban areas, by relative nature, occur on a continuum. Rural and urban residencies are not opposing lifestyles. Rather, they are a rural-urban continuum ranging from living on a remote farm, to a village or small town, to a larger town or city, to a large metropolitan area with a core inner city.DIF: Cognitive Level: KnowledgeREF: Page 390

12. A nurse who is explaining to a client why it is important to take medication states, The medication takes a couple of weeks to be effective, but then you should feel better. When the client is next seen, no medication has been purchased. Which of the following is the most likely explanation? a. The nurse emphasized that eventually the client would feel better, but the client needed to feel better immediately so didnt bother with the drug. b. The medication required a trip to the pharmacy, and the client just hadnt had time to obtain the drug yet. c. The medication was too expensive for the clients family. d. The client really hadnt understood why the medication was important.

ANS: A Although any of the answers given is possible, if we look closely at what the nurse stated, there may have been a cultural disconnect based on time orientation. Many nurses are future oriented, whereas many families may place greater value on quality of life and view present time as being more important. When nurses discuss health promotion and disease prevention strategies with persons from a present orientation, they should focus on the immediate benefits these clients would gain rather than emphasizing future outcomes.

18. A nurse explained to a new mother that because she had tested positive for the hepatitis B virus, her newborn son would need the hepatitis B vaccine immediately and then also an immune globulin injection. Wait, said the new mother. Why is my son getting two shots? Which of the following statements would be the best response by the nurse? a. One injection protects your son, while the other encourages his body to build up immunity. b. One shot keeps your son from getting sick, while the other is a typical vaccine to prevent you from accidentally infecting him. c. Since youve already been infected with the virus, your son needs twice as much protection. d. The second shot is just to make sure the first one works.

ANS: A Because infected persons may not have any symptoms, all pregnant women should be tested for HBsAg. If the mother tests positive, her newborn needs hepatitis B immune globulin to provide passive immunity and thus prevent infection. In addition, the newborn is given the hepatitis B vaccine at birth, with two follow-up injections, to build active immunity to the infection.

16. For a bedridden Muslim patient, the nurse rearranges the room and moves the bed so that it faces toward Mecca for the patients daily prayers. Which of the following is the nurse demonstrating through these actions? a. Accommodation b. Awareness c. Brokering d. Imposition

ANS: A Cultural accommodation involves including aspects of the patients religious beliefs and/or folk practices in the traditional health care system to implement essential treatment plans. For this patient, daily prayer in the tradition of Islam is importantfrom the patients perspective, possibly more important than medical treatment.

4. A nurse is caring for a client of another culture. Which of the following actions would be most appropriate for the nurse to take? a. Alter personal nonverbal behaviors to reflect the cultural norms of the client. b. Keep all behaviors culturally neutral to avoid misinterpretation. c. Rely on friendly gestures to communicate caring for the client. d. Avoid any pretense of prejudice by treating the client in the same way as any other client.

ANS: A Cultural competence in nursing includes adoption of culturally congruent behaviors. Culturally skillful nurses use appropriate touch during conversation, modify the physical distance between themselves and others, and use strategies to avoid cultural misunderstandings while meeting mutually agreed-upon goals. Nurses who strive to be culturally competent respect people from other cultures and value diversity, which helps them to provide more responsive care.

15. At a local hospital, postpartum care policy requires that nurses observe the mother during infant care to assess the mothers ability to care for the new baby and to promote bonding. A new mother expresses concern that in her country, all infant care is provided by other family members so that the mother can rest and recover. Which of the following actions would be taken by a culturally competent nurse? a. Allow family members to provide the newborns care and assess the mothers knowledge of child care through discussion. b. Reinforce the importance of bonding and that all good mothers gladly assume these responsibilities. c. Explain that the process of postpartum recovery does not require this much rest and require that she provide infant care. d. State that she must abide by hospital policy because documentation of the mothers ability to give the infant care is required for discharge.

ANS: A Culturally competent nursing care focuses on the specific patient, reflects the patients individual beliefs and values, and is provided with sensitivity.

6. A school nurse asks a class about the ways HIV can be transmitted. Which of the following comments by a student indicates a need for additional teaching? a. I wouldnt sit next to someone with HIV. b. Having unprotected sex with someone who is infected spreads HIV. c. Sharing needles when shooting up drugs spreads HIV. d. Transfusions of blood products that are contaminated can spread HIV.

ANS: A HIV can be transmitted through exposure to blood, semen, vaginal secretions, and breast milk and by sharing needles, syringes, and other equipment used to prepare injectable drugs. It can also be spread by perinatal transmission from mother to child through delivery or breastfeeding and by transfusions of contaminated blood. HIV is not transmitted through casual contact such as touching or hugging someone who has HIV infection. It is not transmitted by insects, coughing, sneezing, office equipment, or sitting next to or eating with someone who has HIV infection. Worldwide, the largest number of HIV infections result from heterosexual transmission.

1. Which of the following best describes most Americans attitude toward immigrants? a. Ambivalence because there are no clear solutions about how to address their needs b. Strongly negative because immigrants take jobs that native-born Americans could have instead c. Strongly positive because immigrants bring useful job skills and often join previous family members already in the United States d. Strong opposition to further immigration because of the increasing population in the United States

ANS: A Most Americans are ambivalent about immigration, recognizing both the positive and negative aspects involved and realizing that it is a complex issue that has no clear solutions.

19. When teaching a nutrition class to a student group with a large Hispanic population, the school nurse incorporates foods such as salsa and other healthy dishes familiar to Hispanic students into the presentation. Which of the following best describes the action taken by the nurse? a. Primary prevention b. Secondary prevention c. Tertiary prevention d. Both primary and secondary prevention

ANS: A Primary prevention involves activities such as health teaching to prevent a problem from occurring.

10. A male client visits the clinic office complaining of a yellow, green discharge from his penis. Which of the following STDs has the client most likely contracted? a. Gonorrhea b. Syphilis c. Herpes simplex virus 2 d. Human papillomavirus

ANS: A The symptoms for gonorrhea in a male include a burning sensation when urinating, or a white, yellow-green discharge from the penis. Some men may get swollen or painful testicles. In men, gonorrhea can cause epididymitis, a painful condition of the testicles that if untreated can lead to infertility.

13. Which of the following statements best explains why HSV-2 infection is more challenging for a client than gonorrhea infection? a. HSV-2 is a viral infection that is both chronic and incurable. b. HSV-2 is extremely expensive to treat. c. HSV-2, like HIV, is almost impossible to diagnosis in the early stages. d. Once a person has been treated for HSV-2, the person is immune to further outbreaks.

ANS: A Unlike gonorrhea, there is no cure for HSV-2 infection; it is considered a chronic disease.

14. A nurse is providing education to a parent-teacher organization about substance use and abuse among children. Which information would the nurse most likely include in this presentation? a. Inhalants are among the first drugs that young children use. b. Implementing the DARE program is the most effective way to prevent drug-use. c. Underage drinking is not a problem in most communities. d. Early experimentation with substances decreases the likelihood of future abuse.

ANS: A Inhalants are among the first drugs that young children use.

20. Drug education programs in the school, such as Project DARE, are an example of which level of prevention? a. Primary level of prevention b. Secondary level of prevention c. Tertiary level of prevention d. Primary health care prevention

ANS: A Primary prevention includes education about drugs and guidelines for their use. DARE is a drug use prevention program.

23. A nurse caring for adolescents who have experienced substance abuse recognizes that they are most influenced by: a. Family-related factors b. Positive media messages c. Socioeconomic level d. Peer pressure

ANS: A Substance abuse among adolescents is influenced most by family-related factors, such as genetics, family stress, and parenting styles.

27. The nurse collects a urine specimen knowing that urinalysis for drug testing can be used to determine: a. Whether a certain drug has been used in the recent past b. The degree of intoxication c. The extent of performance impairment d. How much alcohol was consumed

ANS: A The nurse collects a urine specimen with the rationale that urinalysis for drug testing can be used to determine whether a certain drug has been used in the recent past.

2. A nurse recognizes that although a patient speaks English, the patient is from a culture with which the nurse is unfamiliar. Therefore, a cultural assessment should be attempted. Which of the following questions should the nurse ask? (Select all that apply.) a.Can you tell me where your family is from? b.Do you practice a particular religious faith? c.Have you ever been in an American hospital before? d.Is there anything special we need to know about your food preferences?

ANS: A, B In a brief cultural assessment, nurses ask clients about their ethnic background, religious preference, family patterns, cultural values, language, education, politics and health practices. Nurses want to also ask about the clients perception of the health issue and what caused it and how it should be treated as well as the results they expect from the care they get. Such basic data help nurses understand the client from the clients point of view and recognize what is unique about the person, thus avoiding stereotyping.

2. A woman and man who have come to the health clinic begin to argue loudly. You gave me an STD! the man yells. The woman screams back, Not me. I dont have an STD! Which of the following statements would be most appropriate for the nurse to say to them? (Select all that apply.) a. Actually, youre very fortunate to have been tested so you and your partner can begin treatment before more serious damage is done. b. Some STDs may not have any symptoms, so you need to be tested for other conditions and treated if necessary. c. Sometimes the test is inaccurate, so before getting too upset, you should ask to be tested again. d. You may be able to get treatment from your pharmacist so you wont have to be embarrassed like this again.

ANS: A, B, C Often cases of gonorrhea and chlamydia are asymptomatic, so treatment may not be sought and these infections are spread to others through sexual activity. Similarly, during latency, syphilis has no symptoms. It should be noted, however, that STD test results can sometimes be incorrect and the coexistence of other medical conditions may cause a false-positive test result. Having a partner retested, if the results were negative, would also suggest retesting the first person.

MULTIPLE RESPONSE 1. Which medical problems would a client with chronic alcohol abuse most likely experience? Select all that apply. a. Gastrointestinal disturbances b. Cardiac dysrhythmias c. Depression d. Organic brain syndrome

ANS: A, B, C, D Gastrointestinal disturbances, cardiac dysrhythmias, depression, and organic brain syndrome are some effects of chronic alcohol abuse.

1. An undocumented immigrant comes to a physicians office to receive care. Which of the following services can the client receive? (Select all that apply.) a. Treatment for tuberculosis b. Treatment for Type 2 diabetes c. Immunization for polio d.Physical examination

ANS: A, C Undocumented immigrants or illegal aliens are individuals who have crossed a border into the United States illegally or whose legal permission to stay in the United States has expired. They are eligible only for emergency medical services, immunizations, treatment for the symptoms of communicable diseases, and access to school lunches.

14. A Buddhist patient enters the hospital for diagnostic testing just before lunch time. The nurse tells the aide to give a meal tray to the new patient, because no tests will be done until later that evening. The aide gives the patient a meal of Salisbury steak, bread, green beans, and potatoes with brown gravy. The patient eats nothing but a slice of bread and the green beans. Which of the following considerations was omitted by the nurse? a. The patient should not be served any food until a physicians order is obtained. b. The patients Buddhist faith probably requires a vegetarian diet. c. The patient may be too frightened about the tests to want to eat very much. d. The patient may have diabetes or be allergic to some foods.

ANS: B Although it is always wise to check with a patient before sending in food, the meal given to this patient was offensive. Most Buddhists are vegetarians and dont eat meat.

8. Which of the following best explains why some health clinics allow clients to be tested for HIV anonymously with no record of the clients name, address, or contact information? a. Client doesnt actually ever have to be told the results of the test. b. Client may be engaged in illegal activities (drug use). c. Client plans on not paying for the test and collection agencies will not be able to harass them. d. Client wants to be sure care providers dont share results with their family.

ANS: B An advantage of anonymous testing may be that it increases the number of people who are willing to be tested, because many of those at risk are engaged in illegal activities. The anonymity eliminates their concern about the possibility of arrest or discrimination.

21. A mother felt very guilty that her baby was born HIV positive. When the nurse suggested the usual DPT and MMR immunizations, the mother was extremely upset. Dont you know HIV children are immunosuppressed? she exclaimed. Which of the following would be the nurses best response? a. All children have to have these immunizations before they can attend school. b. Being HIV positive, your child is more likely to catch an infection and be very ill if not immunized. c. Im so sorry; I forgot for a moment your child was HIV positive. d. The American Pediatric Association requires all health care providers to offer these immunizations to all parents; it is your choice whether or not to accept them.

ANS: B Because of impaired immunity, children with HIV infection are more likely to get childhood diseases and suffer serious consequences of the diseases. Therefore, DPT, IPV, and MMR vaccines should be given at regularly scheduled times for children infected with HIV. Other immunizations may also be recommended after medical evaluation. Many states do have a no shots, no school law, but exceptions can be made. The APA does recommend immunizations for most children, but this is not the best answer.

3. A nurse wishes to develop cultural competence. Which of the following actions should the nurse take first? a. Complete a survey of all the various ethnicities represented in the nurses community. b. Consider how the nurses own personal beliefs and decisions are reflective of his or her culture. c. Invite a family from another culture to join the nurse for an event. d. Study the beliefs and traditions of persons living in other cultures.

ANS: B Cultural awareness requires self-examination and an in-depth exploration of ones own beliefs and values as they influence behavior.

10. A nurse states, The best way to treat a client from another country is to care for them the same way we would want to be cared for. After all, we are all humans with the same wants and needs. What does this statement reflect in relation to culture? a. Awareness b. Blindness c. Knowledge d. Preservation

ANS: B Cultural blindness is the tendency to ignore differences between cultures and to act as if they do not exist. People from different cultures may have different expectations, wants, and needs.

2. A nurse is about to despair. Earlier in the week, she carefully taught a patient from a different culture exactly how much medication to take and emphasized the importance of taking the correct amount. However, the patient is back in the hospital today with symptoms of an overdose, although the patient denies taking more than the label indicated. Which of the following is the most likely explanation? a. The patient was taking more mediation in the hope of getting well faster. b. The patient was also taking folk medicines that had many of the same effects and perhaps some of the same ingredients as the prescribed medication. c. The patient truly did not understand and thought the dose being taken was correct. d. The patient had a unique response to the medication and should have a smaller dose ordered.

ANS: B For fear of disapproval, a person may not tell the nurse that he or she is using folk medicine as well as Western medication. The two medicines may have cumulative effects that could be dangerous to the client. Nurses who lack cultural knowledge may develop feelings of inadequacy and helplessness because they are often unable to effectively help their clients.

15. Which of the following best describes the characteristic appearance of lesions of human papillomavirus (HPV)? a. Solitary growth with elevated borders and a central depression b. Elevated growths with a cauliflower appearance c. Thin-walled pustules that rupture to form honey-colored crusts d. Vesicles that ulcerate and crust within 1 to 4 days

ANS: B HPV causes genital warts that appear as textured surface lesions, with what is sometimes described as a cauliflower appearance. The warts are usually multiple and vary between 1 and 5 mm in diameter.

19. A client with tuberculosis (TB) asks why the nurse is required to watch the client swallow the medication each day. Which of the following statements is the best response by the nurse? a. Clients with TB are often noncompliant, so if I directly observe, you will be sure to take the drugs that have been ordered. b. This therapy is recommended to make sure that you receive the treatment you need and the infection doesnt become resistant to the drugs. c. This is to make sure you take your medication if your condition becomes so advanced that you do not have enough cerebral oxygenation to remember. d. Tuberculosis medications are very expensive so this method ensures that government money doesnt get wasted on those who will not take the drugs.

ANS: B It is important to be respectful to clients and to consider their perspective and psychological health while also responding truthfully. Directly observed therapy (DOT) programs for TB medication involve the nurse observing and documenting individual clients taking their TB drugs. When clients prematurely stop taking TB medications, there is a risk that the TB will become resistant to the medications. This can affect an entire community of people who are susceptible to this airborne disease. DOT ensures that TB-infected clients receive adequate medication. Thus, DOT programs are aimed at the population level to prevent antibiotic resistance in the community and to ensure effective treatment at the individual level. Many health departments have DOT home health programs to ensure adequate treatment.

12. Which of the following best explains why chlamydia is a major focus of public health efforts? a. It has more serious long-term outcomes than other STDs b. It can cause problems in infants born to infected mothers c. It is not frequently seen in the United States d. It is so difficult and expensive to treat

ANS: B Like gonorrhea and other STDs, chlamydia can cause neonatal complications in infants born to infected mothers, although it is rather easily treated with antibiotics. Chlamydia does have serious long-term outcomes for the client, but so do syphilis and other STDs. However, unlike syphilis, which in its later stages is rare in the United States, chlamydia is the most common reportable infectious disease in the United States and hence is a major focus of public health.

20. A nurse is concerned about the prevalence of tuberculosis among migrant farmworkers. Which of the following activities would be best to use when implementing tertiary prevention? a. Administer purified protein derivative (PPD) to contacts of those with tuberculosis. b. Initiate directly observed therapy (DOT) for tuberculosis treatment. c. Provide education about the prevention of tuberculosis to members of the migrant community. d. Use skin tests to screen migrant health workers for tuberculosis infection.

ANS: B Tertiary prevention is carried out among persons already infected with the disease. In this instance DOT ensures compliance with treatment to cure the disease and to prevent worsening or the development of secondary problems.

14. When a nurse discovers that a woman has been treated for cervical cancer, the nurse asks the woman whether she has ever been tested for HIV or other STDs. The woman is offended and asks why the nurse would ask her such a thing. Which of the following statements would be the best response from the nurse? a. Cervical cancer treatments may decrease immunity, so that it is easier to acquire STDs. b. Cervical cancer usually is caused by HPV, and often the presence of one STD is accompanied by other STDs. c. The presence of an STD in women with cervical cancer may lead to congenital defects in offspring. d. The presence of an STD in a woman with a history of cervical cancer has been associated with a relapse of the cancer after treatment.

ANS: B The link between HPV infection and cervical cancer has been established and is associated with specific types of the virus. In 80% to 90% of cases of cervical cancer, evidence of HPV has been found in the tumor. Additionally, HSV-2 infection is linked with the development of cervical cancer. Because the presence of an STD increases the risk for the presence of other STDs, it is essential to screen for this information.

12. The nurse is interviewing a client who reports that he drinks alcohol when snorting cocaine. This client is experiencing: a. Drug addiction b. Polysubstance use c. Drug and alcohol abuse d. Indiscriminate drug use

ANS: B A client who drinks alcohol when snorting cocaine is suffering from polysubstance abuse

17. A nurse is providing care for a family in which a non-addict is experiencing a stress-induced preoccupation with an addicts life leading to extreme dependence on the addict. This describes: a. Obsession b. Codependency c. Enabling d. Addiction

ANS: B Codependency occurs when a person in a close relationship with an addict develops unhealthy coping mechanisms to continue the relationship.

4. Dependence is a term that refers to the: a. Presence and severity of withdrawal symptoms b. Necessary use of drugs to prevent withdrawal symptoms c. Level of intoxication associated with addiction d. Amount of substance needed to satisfy cravings

ANS: B Dependence refers to the necessary use of drugs to prevent withdrawal symptoms.

18. Which stage of change is a client experiencing when a commitment to take action has not yet been made? a. Precontemplation b. Contemplation c. Preparation d. Action

ANS: B During contemplation, the individual is aware that a problem exists and is seriously thinking about overcoming it but has not yet made a commitment to take action. The nurse can encourage the individual to weigh the pros and cons of the problem and the solution to the problem.

5. What has happened in the United States as a result of prohibition? a. Substance abuse has declined. b. Violent crime and corruption has increased. c. The need for community health nurses has declined. d. Facilities to treat substance abuse have improved.

ANS: B Prohibition has led to increased crime and corruption among law officials related to the illicit market.

22. Which action would the nurse take when giving advice using the acronym FRAMES? a. Offer a menu of options or choices. b. Provide clear guidance to change risky behavior. c. Support the clients ability to change the behavior. d. Emphasize the clients responsibility for change.

ANS: B Providing clear guidance to change risky behavior is one of the elements of the FRAMES method of intervention.

16. The leading cause of birth defects in the United States is: a. Cigarette smoking b. Fetal alcohol syndrome c. Marijuana use d. HIV from sharing needles

ANS: B The leading preventable cause of birth defects in the United States is fetal alcohol syndrome, causing mental and behavioral impairment.

3. The public health nurse comes to the hospital to see a client just diagnosed with hepatitis A. The nurse says, Im sorry to bother you when youre not feeling well, but I need to ask you a few questions. Which of the following questions would be most appropriate for the nurse to ask the client? (Select all that apply.) a. Do you know how you got this infection? b. Who lives with you? c. Where are you employed? d. Where do you usually eat?

ANS: B, C The nurse should ask about sexual contacts and ask who (if anyone) lives with the client because the named individuals will need to have immune globulin administered to hopefully prevent the spread of hepatitis A and a community epidemic. The nurse should also ask about the clients place of employment because certain settings warrant special considerations. For example, in restaurants, hospitals, daycare centers, or other institutions, the lack of careful hand-washing by an infected worker can result in contamination of many others.

1. In which of the following cases would the school nurse be correct to advise the parents of an HIV-infected child to keep the child home from school? (Select all that apply.) a. The child develops allergies with sneezing. b.The child persists in biting behavior or is unable to control body secretions. c. The nurse is not comfortable with being responsible for the child. d. There is an outbreak of chickenpox in the school.

ANS: B, D Not attending school may be advisable if cases of childhood infections, such as chickenpox or measles, occur in the school, because the immunosuppressed child is at greater risk for suffering complications. Alternative arrangements, such as homebound instruction, might be instituted if a child is unable to control body secretions or displays biting behavior. HIV-positive children are encouraged to obtain routine immunizations, because their immune systems are compromised and they are more susceptible to such infections. To date, no cases of HIV infections being transmitted in a school setting have occurred in the United States. Strong disapproval by parents of well children is an opportunity to teach these persons how HIV is transmitted.

18. A 40-year-old Bosnian, Muslim woman who does not speak English presents to a community health center in obvious pain. She requests a female health care provider. Through physical gestures, the woman indicates that the pain is originating in either the pelvic or genital region. Which of the following interpreters would be the most appropriate in this situation? a. A Bosnian male who is certified as a medical interpreter b. A female from the clients community c. A female who does not know the client d. The clients 20-year-old daughter

ANS: C Although having experience in medical interpretation is important, in many cultures it is inappropriate to have a male interpreter for females. This client has specifically requested a female provider; therefore, one might anticipate that the client will not be as forthcoming with a male interpreter. Regardless of certification and ability, the interpreter cannot interpret information the client may withhold because she feels it inappropriate to discuss private matters in front of a male. This client may also feel it inappropriate to have private matters interpreted by her daughter (especially if they are of a sexual nature or if they involve infidelity). Additionally, to avoid a breach of confidentiality, the nurse should avoid using an interpreter from the same community as the client.

7. Mexican immigrants who take metamizole (Mexican aspirin) for pain may experience life-threatening agranulocytosis. Which of the following actions would be taken by a nurse who employs cultural repatterning? a. Complete a cultural assessment to identify any other dangerous medications that the client may be taking. b. Put this into perspective by considering that many drugs used in the United States cause agranulocytosis. c. Explain the harmful effects of metamizole and recommend an alternative medication for pain. d. Recognize that taking metamizole is common among persons living in Mexico and accept this as a cultural tradition.

ANS: C Cultural repatterning means that the nurse works with clients to help them reorder, change, or modify their cultural practices when the practice is harmful to them.

9. A nurse is providing education to a client about the use of PrEP. Which of the following statements would the nurse include as part of this teaching? a. Side effects of PrEP include extreme lethargy and joint pain. b. PrEP has been shown to be effective in preventing transmission of the disease from sharing needles. c. The effectiveness of PrEP will depend on your adherence to the medication regimen. d. PrEP will prevent you from contracting HIV and Hepatitis B.

ANS: C Pre-exposure prophylaxis, or PrEP, is a new HIV prevention method for people who do not have the infection but would like to reduce their risk of becoming infected. PrEP requires taking a pill to prevent the HIV virus from getting into the body. It has been shown to be effective for people at very high risk for HIV infection through sex; the results about its effectiveness with injection drug users are not yet available. This prevention method requires strict adherence to taking the medication and having regular HIV testing; it is also used in combination with other HIV prevention methods rather than in isolation (CDC: PrEP, 2012b).

7. In the United States, which demographic group has the highest risk for HIV infection? a. African-American homosexual men b. Hispanic IV drug abusers c. White homosexual men d. White lesbians

ANS: C The largest number of new HIV infections in 2009 (11,400) were in white men who had sex with other men (MSM), and this was followed by black MSM (10,800).

17. A nurse gives detailed information on how to apply for Medicaid to a new mother who moved to the United States from Russia about 10 years ago. The nurses next client is an African-American mother of newborn twins who worked until the children were born. The nurse knows the woman is eligible to maintain her insurance after her employment was lost and does not discuss insurance options at all. Which of the following errors is being made by the nurse? a. Covert intentional prejudice b. Covert unintentional prejudice c. Overt intentional prejudice d. Overt unintentional prejudice

ANS: C The nurse may have assumed that the African-American mother knew the available resources and could negotiate for assistance on her own and that the immigrant Russian woman had no experience negotiating government programs and thus needed the nurse to advocate for her and inform her of the programs available to her. The nurse, not knowing the health-seeking behaviors of either client, stereotyped both women and intentionally used her informational power to help one client while denying assistance to the other client.

2. Which of the following provides the best explanation as to why people do not immediately seek medical treatment when they first become ill with HIV? a. They are afraid to get tested for fear results will be positive. b. They avoid the problem (maybe it will go away). c. They dont recognize their symptoms as possibly being due to HIV. d. It is too expensive to get an HIV test.

ANS: C When HIV enters the body, it can cause a flu-like syndrome referred to as a primary infection or acute retroviral syndrome. This may go unrecognized. The symptoms are similar to flu or a bad cold including sore throat, lethargy, rash, fever, and muscle pain. An antibody test at this stage is usually negative. So the person or a medical provider may not recognize the illness as HIV.

7. A client who is experiencing a drug addiction is at risk for having a(n): a. Infant with a high birth weight b. Easily cured illness or disease c. Accident or committing suicide d. Problem finding employment

ANS: C Alcohol, tobacco, and other drug (ATOD) abuse and addiction is associated with many problems, including neonates with low birth weights and congenital anomalies; accidents, homicides, and suicides; chronic diseases such as cardiovascular disease, cancer, lung disease, hepatitis, human immunodeficiency virus (HIV)/acquired immune deficiency syndrome (AIDS); and mental illness.

19. A nurse is working with a client during a smoking cessation program. The client has developed an action plan for smoking cessation, but has not yet implemented it. Which stage of change is the client experiencing? a. Precontemplation b. Contemplation c. Preparation d. Action

ANS: C During preparation, the individual is prepared for action and may reduce the problem behavior but has not taken effective action yet.

21. When assessing for addiction, the nurse should be aware that a primary symptom of addiction is: a. Anger b. Apathy c. Denial d. Violence

ANS: C In assessing for addiction, the nurse should be aware that a primary symptom of addiction is denial.

2. Which health problem causes more deaths, illnesses, and disabilities than any other in the United States? a. Tobacco addiction b. Alcohol abuse c. Substance abuse d. Caffeine addiction

ANS: C Substance abuse is the number one national health problem.

25. A nurse recommends to a school board that other methods besides the just say no approach be considered when providing drug use prevention education to the elementary students. The rationale for this recommendation is: a. Children do not like to hear the word no, and the approach is too negative. b. Project DARE is more effective and has decreased drug use. c. Children are naturally curious and may experiment with drugs. d. Just say no to drugs does not include smoking and alcohol abuse.

ANS: C The just say no approach does not help young people because children are naturally curious and drug experimentation is often part of normal development, children from dysfunctional homes often use drugs to get attention and escape intolerable environments, and it does not address the powerful component of peer pressure.

1. Why must nurses examine their attitudes about alcohol, tobacco, and other drug abuse and addiction before working with individuals with this health problem? a. Working with clients who have addiction problems often puts the nurse at risk for violence. b. The population of persons with drug and alcohol addiction is found in low-income neighborhoods. c. To be therapeutic, a nurse must develop a trusting non-judgmental relationship with clients. d. Nurses are the primary persons who treat addiction problems.

ANS: C To be therapeutic, the nurse must develop a trusting, non-judgmental relationship with clients. Therefore, nurses must examine their own attitudes ahead of time.

31. A nurse partners with multiple agencies in the community to create a support group for those suffering from addiction in the community. Which level of prevention is being implemented? a. Primary b. Secondary c. Tertiary d. Health promotion

ANS: C When using tertiary prevention, the nurse helps the addict and the addicts family.

4. A student asks the nurse at the student health clinic how AIDS is diagnosed. Which of the following statements would be the best response by the nurse? a. A diagnosis of AIDS is made when a screening test called an enzyme-linked immunosorbent assay (ELISA) is confirmed by the Western blot test. b. A diagnosis of AIDS is made when antibodies to HIV are detected about 6 weeks to 3 months following possible exposure. c. A diagnosis of AIDS is made when antibodies to HIV reach peak levels of 1000/ml of blood. d. A diagnosis of AIDS is made when CD4 T lymphocytes drop to less than 200/ml.

ANS: D AIDS is defined as a disabling or life-threatening illness caused by HIV; it is diagnosed in a person with a CD4 T-lymphocyte count of less than 200/ml with or without documented HIV infection. The HIV antibody test (usually the EIA) is the most commonly used screening test for determining whether the antibody to HIV is present but does not confirm AIDS. Positive results with the EIA are tested further with the Western blot test. However, false-negative results are frequent between 6 weeks and 3 months following exposure.

17. Which of the following statements best explains why many health care providers are more afraid of getting hepatitis B than HIV? a. Everyone would assume the person infected with hepatitis B is a drug user. b. Having HBV would mean no further employment in health care. c. The fatality rate is higher and occurs sooner with HBV. d. There is no treatment for HBV, which can be a very serious illness.

ANS: D Both HBV and HIV are blood-borne pathogens. Health care workers may be exposed to either from needle stick injuries and mucous membrane splashes. However, HBV remains alive outside the body for a longer time than does HIV and thus has greater infectivity. The virus can survive for at least 1 week dried at room temperature on environmental surfaces, and therefore infection control measures are crucial in preventing transmission. There is no treatment for hepatitis B, and although some persons never have symptoms and others fight off the disease, many suffer from chronic hepatitis B, a very serious illness. Others may become chronic carriers of the disease.

11. A family from Mexico comes to the public health department. No one in the family speaks English, and nobody at the health department speaks Spanish. Which of the following actions should be taken by the nurse? a. Attempt communication using an English-Spanish phrase book. b. Call the local hospital and arrange a referral. c. Emphatically state, No hablo Espaol (I dont speak Spanish). d. Obtain an interpreter to translate.

ANS: D Communication with the client or family is required for a careful assessment. When nurses do not speak or understand the clients language, they should obtain an interpreter. The nurse must use strategies that will allow effective communication with the client. The client has the right to receive effective care, to judge whether the care was appropriate, and to follow up with appropriate action if the expected care was not received.

6. The nurse practitioner (NP) discovered that an immigrant client is not taking the penicillin prescribed because his illness is hot and he believes that penicillin, a hot medicine, will not provide balance. Which of the following terms best describes the action taken by the NP when the clients prescription is changed to a different yet equally effective antibiotic? a. Cultural awareness b. Cultural brokering c. Cultural knowledge d. Cultural skill

ANS: D Cultural skill involves the provision of care that is beneficial, safe, and satisfying to the client. The medication change allows the client to retain his cultural beliefs and also satisfies the nurse practitioners need to prescribe an effective antibiotic.

5. A male nurse had a habit of sitting with the lower part of one leg resting over the knee of his opposite leg when collecting a clients history. He stopped doing this around Muslim clients after being told that Muslims were offended when he exposed the sole of his foot (shoe) to their face. Which of the following was exhibited by the nurse when he changed his behavior? a. Cultural accommodation b. Cultural imposition c. Cultural repatterning d. Cultural skill

ANS: D Cultural skill is the effective integration of cultural knowledge and awareness to meet client needsin this case, the clients need to not be offended by having the bottom of the nurses foot or shoe in view of the clients face. The nurse using cultural skill makes sure nonverbal communication techniques take into consideration the clients use of body language and space. Cultural accommodation involves negotiation with clients to include aspects of their folk practices with the traditional health care system to implement essential treatment plans. Cultural imposition is the process of imposing ones values on others. Cultural repatterning is working with clients to make changes in their health practices if cultural behaviors are harmful or decrease their well-being.

22. A nurse was reading PPD tests 24 hours after another nurse had administered them. Which of the following findings would cause the nurse to interpret the test as positive? a. 15 mm of erythema in a client with HIV infection b. 5 mm of induration in an immigrant from a country where TB is endemic c. A 5-mm ruptured pustule with purulent drainage in a homeless client d. 10 mm of swelling and increased firmness in a client recently released from a correctional facility

ANS: D Erythema alone does not indicate a positive finding. For a PPD test to be positive, induration (swelling with increased firmness) must be present. A diameter of 10 mm induration would be a positive finding in an immigrant from a region with high TB infection. A small pustule in a homeless client undoubtedly is an infection but may not be due to the PPD test.

5. The correctional health nurse is doing a quick assessment on a newly admitted inmate who is HIV positive. Which of the following diseases should the inmate receive screening for immediately? a. Herpes zoster b. Hepatitis B c. Hepatitis C d. Tuberculosis

ANS: D HIV-infected persons who live near one another, such as in correctional facilities, must be carefully screened and deemed noninfectious before admission to such settings. A person with HIV is more susceptible to opportunistic infections, the most common of which is TB.

11. A client is being treated for secondary syphilis. Which of the following signs and symptoms would the nurse anticipate the client would exhibit? a. Chancre at the site of entry b. Jaundice c. Difficulty coordinating muscle movements d. Skin rash without itching

ANS: D Secondary syphilis occurs when the organism enters the lymph system and spreads throughout the body. Signs include skin rash on one or more areas of the body and do not cause itching. Other symptoms may include fever, swollen lymph glands, sore throat, patchy hair loss, headaches, weight loss, muscle aches, and fatigue.

8. A health care worker tells a nurse, It does no good to try to teach those Medicaid clients about nutrition because they will just eat what they want to no matter how much we teach them. Which of the following is being demonstrated by this statement? a. Cultural imposition b. Ethnocentrism c. Racism d. Stereotyping

ANS: D Stereotyping occurs when someone attributes certain beliefs and behaviors about a group to an individual without giving adequate attention to individual differences. In this instance, the health care worker makes the assumption that clients with low incomes are not educable. The health care worker is guilty of making another assumption as well: noncompliance among other Medicaid clients the worker has known may have been related to an inability to afford nutritious food.

23. A high school student is planning to volunteer at the hospital after school, so she needs to have a Mantoux test before beginning. Which of the following information should the nurse provide to the new volunteer? a. I will be using tiny tines to administer the TB antigen to the skin on your arm. b.Notify the clinic immediately if you experience any redness or itching at the test site. c. The areas should be kept dry until you return; cover it with plastic wrap when bathing. d. You will need to return in 2 to 3 days to have any reaction interpreted.

ANS: D The Mantoux test is a TB skin test that involves a 0.1-ml injection containing 5 tuberculin units of PPD tuberculin (not tines as in the TB tine test). The site should be examined for a reaction 48 to 72 hours (2 to 3 days) after injection. Only induration should be measured, and the results should be recorded in millimeters.

16. A client who is very upset says to the nurse, But we always used a condom! How could I have genital warts? Which of the following would be the best response by the nurse? a. Are you positive you always used a condom? b. Condoms dont always work. c. The condom might have had a tear in the latex. d. Skin to skin contact to a wart may have occurred outside the area that the condom covers.

ANS: D The challenge of HPV prevention is that condoms do not necessarily prevent infection. Warts may grow where barriers, such as condoms, do not cover, and skin-to-skin contact may occur.

3. A client was clearly very relieved when an HIV test came back negative. Thank goodness. Ive had sex several times without a condom, and when one of my friends said he was sick, I think I panicked. Which of the following would be most important to emphasize to the client immediately? a. Abstinence is the only way to be certain you are HIV-free. b. Sex should be restricted to one partner. c. The test could be wrong and the client might still have an HIV infection. d. The test would not cover any recent infection, so if the client has had recent unprotected sex, the test should be repeated in 3 months.

ANS: D The client needs to understand that the test cannot identify infections that may have been acquired within the previous 3 months before the test. Appearance of the HIV antibody can take up to 12 weeks. All persons who have an STD test should be counseled about risk-reduction activities before and after the test is done.

30. A nurse refers a client to Alcoholics Anonymous (AA). Who will play the most important role in the treatment of alcoholism at this support group? a. Self b. Spouses c. Children d. Peers

ANS: D Alcoholics Anonymous began a strong movement of peer support to treat a chronic illness. The fellowship, support, and encouragement among AA members provide a vital social network for the person recovering from an addiction.

15. Which individual would mostly likely have the lowest blood alcohol concentration? a. An individual who consumes a drink with a high concentration of alcohol b. An individual who drinks without consuming food c. An individual of the female gender d. An individual with a high body weight

ANS: D An individual with a high body weight would likely have the lowest blood alcohol concentration. Increased alcohol concentration, drinking without consuming food, and being a female are all factors which increase blood alcohol concentration.

29. When assessing drug use patterns of a client, which question is most appropriate for the nurse ask? a. Where was the drug acquired? b. Who gave the drug to you? c. What drug was taken? d. How often do you use the drug?

ANS: D During an assessment, all relevant drug-use history is collected and aids in the assessment of drug-use patterns. The nurse should note any changes in drug-use patterns over time. Thus, the most appropriate question is the nurse asking how often a client uses drugs.

26. When providing comprehensive education about HIV transmission for an injection drug user (IDU), the most important information to include is: a. How and where to acquire needles b. Proper needle usage and possible injection sites c. Spreading of the virus via homosexual activities d. Using bleach between needle uses

ANS: D IDUs represent the most rapidly growing source of new cases of acquired immunodeficiency syndrome (AIDS), and they are the greatest risk for spread of the virus in the heterosexual community. Emphasis is being placed on reduction of the transmission of this disease through contaminated needles. Nurses should provide education on cleaning needles with bleach between uses and on needle exchange programs to decrease the spread of the virus.

10. The most widely used illicit drug in the United States is: a. Alcohol b. Heroin c. Cocaine d. Marijuana

ANS: D Marijuana is the most widely used illicit drug in the United States.

13. Which physical effect is the nurse most likely to be concerned about when caring for a client who has had long-term marijuana use? a. Constipation b. Cardiac dysrhythmia c. Abdominal pain d. Respiratory tract damage

ANS: D Respiratory tract damage occurs from smoking the drug and is the greatest physical concern among chronic users.

28. What would a nurse look for when investigating the setting of a particular drug experience? a. A back alley or abandoned building where people are using crack b. An area at the local high school where students are known to smoke cigarettes c. The individuals using the drug, including that individuals expectations d. The physical, social, and cultural environment within which the use occurs

ANS: D Setting is the influence of the physical, social, and cultural environmental within which the use occurs. The other answers are too specific; the last option covers the entire scope of the setting.

virtues

Acquired traits of character that dispose humans to act in accord with their natural good

Enabling

Act of shielding or preventing the addict from experiencing the consequences of the addiction. It also applies to shielding individuals from the consequences of their actions more generally.

suicide

Act or an instance of taking your own life voluntarily and intentionally.

Adoption

Action of taking a child by choice into a relationship; to voluntarily as one's own child.

The public health nurse is most likely to use what types of surveillance systems? (Select 2 that apply.)

Active. Passive.

APEXPH goal

Addressed the three core competencies of public health: assessment, assurance and policy development

local public health agencies

Agencies responsible for implementing and enforcing local, state, and federal public health codes and ordinances and providing essential public health programs to a community

referral resource

Agency or source in the community with whom nurses communicate and to which clients are sent for assistance

D

An employer provides a migrant farm family the day off to visit the health clinic in a nearby community and tells them to take all of the time they need. However, the family arrives at the clinic appearing very stressed. In addition to the health issue, which of the following would most likely be a fear experienced by the family? a. Their personal belongings may be stolen while they are at the clinic. b. Immigration officials will send them back to their home country. c. The clinic personnel will look down on them and be biased against them. d. They weren't getting paid for that day, and continued employment is never certain.

C

An undocumented migrant farmworker has been diagnosed with tuberculosis (TB). The local health department initiates treatment by dispensing the first month's supply of medication and educates the client on the need to continue treatment for 6 to 12 months. A major challenge that the client may face related to ongoing treatment for TB is: A. affordable care. B. discrimination. C. fragmented services. D. language barriers.

Formative evaluation

Any program being programmed; is our parts of our plan being worked out? Have necessary resources; happens beginning

principlism

Approach to problem solving in bioethics that uses the principles of respect for autonomy, beneficence, nonmaleficence, and justice as the basis for organization and analysis

consequentialism

Approach whereby the right action is the one that produces the greatest amount go good or the least amount of evil in a given situation

D

At a county board meeting, a nurse reports statistics on drug use in the school. The nurse then requests funding for an after-school recreation program that promotes age-appropriate fun activities and sports. Which of the following objectives is the nurse attempting to meet through this action? a. Help prevent obesity and boredom. b. Offer alternatives to being "on the streets." c. Prevent children from getting in trouble before their parents get home from work. d. Promote healthy lifestyles with physical activity.

D

At a town meeting with public health officials to discuss a communicable disease outbreak, a nurse is asked to explain what is meant by the phrase "a virulent organism." The nurse explains that this means the organism causing the disease is able to do which of the following? a. Bypass normal immunological response mechanisms b. Invade major organ systems c. Produce toxins and poisons that weaken the body d. Produce very severe physical reactions

10. What condition of the rural environment provides increased opportunities for teaching? a. Because people primarily work on family farms, they visit neighbors more often. b. Involvement in rural community activities provides more contact with community residents than in urban areas. c. Nursing responsibilities include activities of primary, secondary, and tertiary prevention. d. The increased illnesses and injuries of rural residents require that they see nurses more often.

B

11. How are firearm injuries different in rural areas as compared with urban areas? a. In rural areas, firearm deaths occur more frequently because of purposeful rather than accidental shooting. b. In rural areas, firearm injuries occur more frequently in the home. c. In rural areas, firearm injuries occur more frequently during official hunting season, whereas the frequency doesn't vary in urban areas. d. In rural areas, firearm injuries are more frequently fatal than in urban areas.

B

3. When using the health measure of death rates for working-age adults, the nurse could expect to find the highest rates in which areas? A. Large metropolitan areas B. Most rural and highly populated urban areas C. Most rural and suburban areas Incorrect D. Small suburban and all urban areas

B

6. What behavior would most likely lead to a positive interaction for the nurse working with Mexican immigrants? a. Avoiding touching the client except when necessary as part of the physical examination b. Calling the client by name, socializing before addressing the problem, and being very respectful c. Keeping all interactions direct, to the point, and targeted on the reason for presentation d. Maintaining a nonconfrontational relationship by avoiding any disagreement even if the nurse does disagree with what the client is saying

B

9. A district health nurse is assigned to two rural counties in the state. To achieve the best outcomes possible in reducing the health disparities for the large number of frail older clients in the two counties, the nurse should consider using what community-oriented nursing skill? A. Assessment B. Case management C. Geriatrics D. Tertiary prevention

B

9. What is noteworthy about many adolescent males picked up for driving without a license in the countryside? a. Most had driver's education classes but haven't yet earned the money to pay for their driver's license test. b. Most were Hispanic or black from a rural area. c. Most were practiced in driving large farm machinery and were just going from one section of the farm to another section. d. Most were practicing their driving skills with their parent in the car.

B

15. A nurse who cares for Mexican migrant farmworkers must be certain to assess for which of the following? Select all that apply. a. Cholera b. Hepatitis c. High blood level of lead d. Malaria e. Tuberculosis f. Yellow fever

B,C,E

10. Rural residents appear to have a more persistent, endemic level of depression. The factors that may contribute to this level of depression may be related to which of the following? (Select all that apply.) A.Crisis intervention B.Gaps in continuum of mental health services C.Sufficient number of mental health services D.Tolerance for destructive coping behaviors E.Trust in the health care professionals

B,D

A district health nurse is assigned to two rural counties in the state. To achieve the best outcomes possible in reducing the health disparities for the large number of frail older clients in the two counties, the nurse should consider using what community-oriented nursing skill? A. Assessment B. Case management C. Geriatrics D. Tertiary prevention

B. Case management

A district health nurse is assigned to two rural counties in the state. To achieve the best outcomes possible in reducing the health disparities for the large number of frail older clients in the two counties, the nurse should consider using what community-oriented nursing skill? A. Assessment B. Case management C. Geriatrics D. Tertiary prevention

B. Case management Nurses working in rural areas, including those working with migrant farmworkers, have opportunities to use many skills of nursing in community health. One of the first and most important is that of prevention. Given the barriers to receiving health care in rural areas, the ideal situation is to prevent health disruptions whenever possible. Case management and community-oriented primary health care are two effective models to address some of those deficits and resolve rural health disparities.DIF: Cognitive Level: ApplicationREF: Page 403

The community health nurse performs an assessment of violence by observing which of the following community characteristics? (Select all that apply.) A. Presence of social support networks B. Crime rates C. Levels of unemployment D. Presence of physical disabilities in individuals E. Presence of family violence

B. Crime rates C. Levels of unemployment

Rural residents appear to have a more persistent, endemic level of depression. The factors that may contribute to this level of depression may be related to which of the following? (Select all that apply.) A. Crisis intervention B. Gaps in continuum of mental health services C. Sufficient number of mental health services D. Tolerance for destructive coping behaviors E. Trust in the health care professionals

B. Gaps in continuum of mental health services D. Tolerance for destructive coping behaviors

Rural residents appear to have a more persistent, endemic level of depression. The factors that may contribute to this level of depression may be related to which of the following? Select all that apply. A. Crisis intervention B. Gaps in continuum of mental health services C. Sufficient number of mental health services D. Tolerance for destructive coping behaviors E. Trust in the health care professionals

B. Gaps in continuum of mental health services D. Tolerance for destructive coping behaviors There appears to be a more persistent, endemic level of depression among rural residents. Factors that relate to this level of depression are high rate of poverty, economic difficulties, economic recession, geographic isolation, insufficient number of mental health professionals, delays in seeking treatment, tolerance of destructive coping behaviors, lack of trust in mental health professionals, and gaps in the continuum of mental health services.DIF: Cognitive Level: KnowledgeREF: Page 394

A hospice nurse is working with a cancer client and his family. The client's 7-year-old son has developed recent school and peer problems. Understanding the risk for disruption of normal development, the nurse plans to first screen the child for: A. developmental disorders. B. mental health problems. C. parental neglect. D. violence potential.

B. mental health problems.

Gonorrhea

Bacterium that infects the mucous membranes of the genitourinary tract, rectum, and pharynx Transmitted through genital-genital contact, oral-genital contact, and anal-genital contact Uncomplicated or complicated Southeast United States has consistently high rates Number of antibiotic-resistant cases in United States has risen

respect for autonomy

Based on human dignity and respect for individuals and allows them to choose those actions and goals that fulfill their life plans unless those choices result in harm to another

Low birth weight

Birthweight of less than 5 1/2 pounds

bioethics

Branch of ethics that applies the knowledge and processes of ethics to the examination of ethical problems in health care

ethics

Branch of philosophy that includes both a body of knowledge about the moral life and a process of reflection for determining what persons ought to do or be, regarding this life

1. What are health professional shortage areas (HPSAs)? a. Areas with inadequate health care facilities for residents b. Isolated areas of underserved populations within urban regions c. Regions with insufficient numbers of health care providers d. Rural regions of the United States with population densities of less than 10,000

C

10. When discussing lifestyle habits, what habits are particularly prevalent in rural areas (more so than in urban areas) of the South? a. Falls and motor vehicle accidents b. Less use of seatbelts and more drinking of alcohol c. Obesity and sedentary lifestyle d. Suicide and homicide

C

12. What group of persons is the fastest-growing component of the rural population? a. Disabled adults b. Elderly women c. Hispanic children d. Retirees

C

18. How is working on a farm different from working in a factory? a. Factories have more dangerous dust than farming dust. b. Farm chemicals are more immediately fatal than most industrial chemicals. c. The work site and the home are the same. d. Workers on farms, unlike those in factories, can bring dangerous substances home on their clothing, which expose their families.

C

2. What is the most accurate description of a migrant farmworker? a. A person who does farmwork as the primary means of employment, although other work may be done when the seasonal work ends b. A person who immigrates to the United States to "follow the crops" in performing seasonal farmwork c. A person who moves from place to place to earn money performing seasonal agricultural work d. A person who specializes in the development of rural land for the purpose of farming

C

2. What is the reason there are so few hospitals in rural areas? a. Few nurses and physicians enjoy the rural lifestyle. b. Historically, hospitals began in large urban areas and have slowly expanded outward but haven't yet reached all rural areas. c. Rural areas have very low population density. d. Rural residents don't trust hospitals, preferring to receive care from neighbors and friends.

C

27. How does the federal government try to both encourage and support rural health clinics under the community health centers program? a. Allowing tax deductions for charitable contributions to rural health clinics b. Donating equipment and supplies to establish such clinics c. Reimbursing both Medicare and Medicaid at a higher rate than otherwise d. Requiring recipients to serve in a rural health clinic for 3 years in exchange for financial assistance while in school

C

28. In what way are Migrant Health Clinics under the Migrant Health Program often different from other clinics? a. Care is restricted to those who can document that they are employed in agriculture. b. They are much better funded than typical rural clinics. c. Staff are typically bilingual and bicultural. d. They welcome illegal immigrants to receive care.

C

4. Within a state, counties designated as Health Professional Shortage Areas (HPSAs) tend to have a high proportion of racial minorities and fewer specialists. This factor may explain conflicting data within a state related to: A. adult immunization rates. B. chronic respiratory illness rates. C. maternal/infant morbidity rates. D. obesity rates.

C

5. A migrant farmworker brings his daughter into the clinic with severe heat stroke from being out in the sun. The nurse explains the danger signs and stresses staying cool and drinking lots of water. The man seems to know this already. So what is the most likely reason this happened? a. It was a rare occurrence, which probably won't be repeated b. The daughter either disobeyed her father and went out to play in the sun or just did not realize how hot she was. c. Children may work on small farms because the family may need the additional income. d. The parents were busy working and didn't realize child was outside so long.

C

6. The local hospital emergency department has recently experienced an increase in gastroenteritis cases among migrant farmworkers. The local health department is informed of this rise in cases and schedules a case mapping of local: A. bars frequented by migrant workers. B. farm fields employing migrant workers. C. housing for migrant workers. D. restaurants frequented by migrant workers.

C

7. An undocumented migrant farmworker has been diagnosed with tuberculosis (TB). The local health department initiates treatment by dispensing the first month's supply of medication and educates the client on the need to continue treatment for 6 to 12 months. A major challenge that the client may face related to ongoing treatment for TB is: A. affordable care. B. discrimination. C. fragmented services. D. language barriers.

C

8. A school nurse is asked to assess a 13-year-old child because of excessive drowsiness and inattention in class. The nurse determines that this is a child of migrant workers on a local produce farm. Based on the nurse's knowledge of migrant worker families, the nurse should first explore the child's potential involvement in: A. delinquent behavior. B. drug use. C. field work. D. Migrant Education Program.

C

It is estimated that one in 15 males becomes a father during his teen years. The nurse should be aware that many young men facing paternity have specific challenges such as: (select all that apply) A. acting as though they are interested when they are disinterested. B. avoiding prenatal care involvement. C. desiring and needing to be involved with their children. D. being rejected by the young woman's family. E. rejecting their role as a father.

C. desiring and needing to be involved with their children. D. being rejected by the young woman's family.

A father brings his stepdaughter to the family clinic for an immunization update before the new school year. The nurse notices the interaction between the young girl and her stepfather. The child appears tense and cautious and wraps her arms around herself in a protective manner. The child startles when touched by the stepfather and pulls away. The stepfather is overheard saying, "I'll leave you here if you don't behave and act nice." In this scenario, it would be important for the nurse to explore in her assessment the possibility of: A. child neglect. B. Family secrets. C. father-daughter incest. D. impaired family functioning.

C. father-daughter incest.

A school nurse is asked to assess a 13-year-old child because of excessive drowsiness and inattention in class. The nurse determines that this is a child of migrant workers on a local produce farm. Based on the nurse's knowledge of migrant worker families, the nurse should first explore the child's potential involvement in: A. delinquent behavior. B. drug use. C. field work. D. Migrant Education Program.

C. field work.

A school nurse is asked to assess a 13-year-old child because of excessive drowsiness and inattention in class. The nurse determines that this is a child of migrant workers on a local produce farm. Based on the nurse's knowledge of migrant worker families, the nurse should first explore the child's potential involvement in: A. delinquent behavior. B. drug use. C. field work. D. Migrant Education Program.

C. field work. Children of migrant farmworkers may need to work for the family's economic survival. Federal law does not protect children from overworking or from the time of day they work outside of school. Therefore children may work until late in the evening or very early in the morning every day of the week. These children may experience constant fatigue and are set up for failure in school.DIF: Cognitive Level: ApplicationREF: Pages 399-400

An undocumented migrant farmworker has been diagnosed with tuberculosis (TB). The local health department initiates treatment by dispensing the first month's supply of medication and educates the client on the need to continue treatment for 6 to 12 months. A major challenge that the client may face related to ongoing treatment for TB is: A. affordable care. B. discrimination. C. fragmented services. D. language barriers.

C. fragmented services. Although migrant workers move from job to job, their health care records typically do not go with them. This leads to fragmented services in such areas as TB treatment, chronic illness management, and immunization. When migrant farmworkers move, they must independently seek out new health services to continue their medications. Also, people with TB may forfeit treatment because they are afraid of immigration authorities.DIF: Cognitive Level: ComprehensionREF: Page 399

The local hospital emergency department has recently experienced an increase in gastroenteritis cases among migrant farmworkers. The local health department is informed of this rise in cases and schedules a case mapping of local: A. bars frequented by migrant workers. B. farm fields employing migrant workers. C. housing for migrant workers. D. restaurants frequented by migrant workers.

C. housing for migrant workers.

The local hospital emergency department has recently experienced an increase in gastroenteritis cases among migrant farmworkers. The local health department is informed of this rise in cases and schedules a case mapping of local: A. bars frequented by migrant workers. B. farm fields employing migrant workers. C.housing for migrant workers. D. restaurants frequented by migrant workers.

C. housing for migrant workers. Housing conditions greatly vary among states and localities. When housing costs are high, 50 farmworkers may live in one house, or three families may share one trailer. Some may live in cars or tents if necessary. Housing may lack individual sanitation, bathing, or laundry facilities. Poor-quality and crowded housing can contribute to such health problems as tuberculosis (TB), gastroenteritis, and hepatitis.DIF: Cognitive Level: KnowledgeREF: Page 398

A nurse new to the community evaluates the resources available to a father that has sought help with his escalating abuse and threats of violence to his family. After making the referral, the nurse approaches the local newspaper about running a series on the nature and extent of human abuse in the community. This strategy would: A. advocate for government programs to treat survivors. B. demonstrate the provider's commitment to address the need for services. C. increase awareness of community resources to address violence and abuse. D. increase the number of individuals identified as perpetrators.

C. increase awareness of community resources to address violence and abuse.

Within a state, counties designated as Health Professional Shortage Areas (HPSAs) tend to have a high proportion of racial minorities and fewer specialists. This factor may explain conflicting data within a state related to: A. adult immunization rates. B. chronic respiratory illness rates. C. maternal/infant morbidity rates. D. obesity rates.

C. maternal/infant morbidity rates.

A nurse in community health is working in a rural setting. In planning for programs to address the population's needs, the nurse should be aware that, in general, rural populations: A. engage in physical activity during leisure time. B. engage in preventive behaviors. C. perceive their overall health as less favorable. D. use seat belts.

C. perceive their overall health as less favorable.

A nurse in community health is working in a rural setting. In planning for programs to address the population's needs, the nurse should be aware that, in general, rural populations: A. engage in physical activity during leisure time. B. engage in preventive behaviors. C. perceive their overall health as less favorable. D. use seat belts.

C. perceive their overall health as less favorable. In general, people in rural areas have a poorer perception of their overall health and functional status than do urban residents. Rural residents older than 18 years assess their health status less favorably than do urban residents. Studies show that rural adults are less likely to engage in preventive behavior, which increases their exposure to risk.DIF: Cognitive Level: KnowledgeREF: Page 392

Community health report card

Community health profiles, needs assessments, scorecards, quality of life indicators, health status, progress reports. Useful tool to help identify areas where change is needed, to set priorities for actions, and to track changes in population over time.

Codependency

Condition characterized by preoccupation and extreme dependency (emotionally, socially, and sometimes physically) on a person. Eventually this dependence on another person becomes a pathological condition that affects the person in all of his or her relationships.

Fetal alcohol syndrome (FAS)

Condition that may occur when a woman has consumed alcohol regularly during pregnancy (about six drinks per day). Infants tend to be of low birth weight and intellectually disabled and may have behavioral, facial, limb, genital, cardiac, or neurological impairments.

An elementary classroom of 28 students has had 13 children who have developed chicken pox over the last 3 weeks. What is the pattern of occurrence?

Continuous source

What is the Nurse's Role in Providing Preventive Care for Communicable Diseases?

Counselor Educator - Safer sex - Drug use Advocate Case manager Primary care provider Community outreach Standard precautions

A recent movie release portrays a criminal as a black female drug user whose abusive boyfriend has two children by different women. She lives in the riot-torn inner city of a large metropolitan area. This best demonstrates what type of factor that influences poverty? Cultural Environmental Political Social

Cultural Cultural attitudes are the beliefs and perspectives that a society values. Perspectives about individual responsibility for health and well-being are influenced by the prevailing cultural attitudes. The media communicate thoughts and attitudes through literature, film, art, television, and newspapers.DIF: Cognitive Level: ApplicationREF: Page 409

A nurse exclaims proudly to other nurses at the agency, "The care I provide is the same for everyone. I treat everyone the same." In fact, this nurse is demonstrating what inhibitor to developing cultural competence?

Cultural blindness

1. How can you explain how the rural population could be growing when young persons from rural areas consistently move to urban areas seeking employment? a. Adults engaged in all the physically demanding farming tasks have longer life spans than people in urban areas. b. Farm families continue to have a higher birth rate than urban families. c. Many formerly migrant worker Hispanics are settling in rural counties. d. Retirees and other urbanites able to conduct business through telecommunication and travel are moving in.

D

11. How could a nurse engage in tertiary prevention related to pesticide exposure? a. Observe farmworkers for evidence of unsafe handling of pesticides b. Provide teaching on how to handle pesticides to avoid or decrease exposure c. Teach farmworkers how to recognize signs and symptoms of pesticide poisoning d. Treat a client who has pesticide exposure to prevent complications

D

13. A migrant farmworker's family seemed to be very stressed, although their employer had given them the day off to visit the clinic and told them to take all the time they needed. In addition to the health issue, what might be the family's fear? a. Their personal belongings may be stolen while they are at the clinic. b. Immigration officials will send them back to their home country. c. The clinic personnel will look down on them and be biased against them. d. They weren't getting paid for that day, and continued employment is never certain.

D

14. What employment opportunities are also among the most dangerous? a. Food processing, textiles, and transportation b. Highway construction, ranching, and public utilities c. Large mechanized factory positions d. Mining, forestry, and fishing

D

15. As the only public health nurse in a rural health clinic, on the basis of risk, who would be your priority group for care? a. African-American and Native American adults b. Adolescents c. Elderly women who live alone d. Migrant farm workers' children

D

17. What is the most frequent cause of farm fatalities and nonfatal injuries? a. Manure pits (methane poison) b. Power take-off augers c. Storage silos d. Tractors e. Very old and barely functioning trucks

D

20. How would you describe migrant farm workers to a friend who is trying to understand why such workers have cultural barriers to receiving care? a. As illegal immigrants, they are not eligible for tax-paid health care. b. Families who travel in isolation seeking agricultural work and don't mix with their employers or other members of the community c. Mexicans who speak primarily Spanish so can't understand English d. Usually perceived as outsiders, they are from many countries and cultures.

D

23. As there are few health care professionals in rural areas, what are many communities choosing to do? a. Advertise that there are more men than women in many rural areas to attract female nurses who want to be wives and mothers b. Offer free housing and office space to a professional who will come to live in the community c. Pay extremely high salaries to professionals willing to relocate d. Support local students obtaining an education while continuing to live at home

D

24. What difference is found in parish nursing in a rural area as opposed to parish nursing in an urban area? a. Rural parish nurses are less likely to be involved in case management or care coordination than their urban counterparts. b. Rural residents are more grateful for the nurse's assistance because there are so few health resources in rural areas. c. Rural residents are typically less committed to traditional values or a strong religious faith. d. Rural residents may discuss health concerns with the nurse wherever they meet, whether in church or the local market.

D

25. A nurse saw there was a need for a hospice facility in a rural area, as so many of the residents were quite elderly and death was common. Why would the community not immediately see the wisdom of the nurse's suggestion? a. Change is always difficult; the nurse just needs to keep reinforcing the need for such a facility. b. The nurse didn't tell residents how much their taxes would need to go up to support such a facility. c. Residents were fearful of confronting death in such an open way. d. Rural residents prefer to use their informal social support networks when help is needed.

D

3. From what cause are rural residents twice as likely to die in comparison with urban residents? a. Chronic diseases, especially cardiac problems and cancers b. Diabetes from the higher incidence of obesity c. Respiratory diseases (from pesticides and herbicides) d. Unintentional injuries

D

4. What fact would cause a geographic area to be called a frontier area? a. Arbitrary distinction recognizing the Great Plains, Intermountain areas, and Alaska b. Geographically located in the West, specifically the Great Plains states c. High death rate from use of guns (homicide or suicide) d. Population of fewer than six persons per square mile

D

8. What must be true for a county to be defined as having persistent poverty? a. At least 10% of the population has been in poverty for at least the last 10 years. b. At least 10% of the population has been in poverty for at least the last 20 years. c. At least 20% of the population has been in poverty for at least the last 20 years. d. At least 20% of the population has been in poverty for at least the last 30 years.

D

A nurse in community health conducts quarterly mental health-promotion and depression-screening programs at the local senior center. The nurse is aware that older adults are at increased risk for developing depression. Using such an intervention also addresses the older adults': A. dependence on their primary care provider. B. normal sensory losses. C. reduced social contacts. D. underutilization of the mental health system.

D. underutilization of the mental health system.

B

During a group counseling session for perpetrators of intimate partner violence, which of the following statements made by a client indicates a lack of insight into his violent behavior? a. "I have been taking out my frustrations with work on my girlfriend." b. "I love my girlfriend and didn't want to hurt her; it was an accident." c. "It might be a good idea for me to temporarily leave the house when I feel myself becoming angry." d. "When I drink alcohol, I become more abusive toward my girlfriend."

D

During an outbreak of hepatitis A, nurses are giving injections of hepatitis A immunoglobulin to selected susceptible persons. Which of the following best describes the type of immunity that will follow the administration of these injections? a. Active immunity b. Long-lasting immunity c. Natural immunity d. Passive immunity

19. A woman stumbled into the rural health clinic and explained that she had an incredible headache. She was sweating and began vomiting into the wastebasket by the intake nurse's desk. What might you conclude? a. Acute adult panic disorder b. Acute alcohol intoxication c. Acute appendicitis d. Acute meningitis e. Acute pesticide poisoning

E

What are the core competencies of the nurse participating in surveillance and investigation activities?

Effective communication skills. Ability to collaborate with community partners. Leadership and systems thinking.

Deinstitutionalization

Effort to move long-term psychiatric patients out of the hospital and back into their own community.

A nurse in community health conducting a home visit notices a 4-year-old girl sitting on a stool in an adjoining room. The girl is quiet and withdrawn, rarely makes eye contact, and does not leave the room. The nurse proceeds to ask about the child and attempts to engage the child in conversation. The nurse is assessing for what indicators of child abuse? Emotional abuse Emotional neglect Physical abuse Physical neglect

Emotional neglect. Neglect is more difficult to assess than is abuse. Emotional neglect is the omission of basic nurturing, acceptance, and caring essential for healthy personal development. These children are largely ignored or in many cases treated as a nonperson. It is difficult for a neglected child to feel a great deal of self-worth because the parents have not demonstrated that they value the child. Astute observations of children, their homes, and the way they relate to their caregivers can provide clues of neglect.DIF: Cognitive Level: ApplicationREF: Page 464

Setting

Environment—physical, social, and cultural—as a variable determining a person's reaction to a drug.

beneficence

Ethical principle that is complementary to nonmaleficence and requires that we "do good" and prevent or avoid doing harm. We are limited by time, place, and talents in the amount of good we can do. We have general obligations to perform those actions that maintain or enhance the dignity of other persons whenever those actions do not place an undue burden on health care providers.

utilitarianism

Ethical theory based on the weighing of morally significant outcomes or consequences regarding the overall maximizing of good and minimizing of harm for the greatest number of people

deontology

Ethical theory that bases moral obligation on duty and claims that actions are obligatory irrespective of the good or bad consequences that they produce. Because humans are rational, they have absolute value. Therefore, persons should always be treated as ends in themselves and never only as means

Formative evaluation

Evaluate the progress of a program

Set

Expectation, including unconscious expectation, as a variable determining a person's reaction to a drug.

What is/isnt HIV transmitted by?

Exposure to blood, semen, vaginal secretions, and breast milk HIV is not transmitted through casual contact such as touching or hugging someone with HIV infection. It is not transmitted by insects, coughing, sneezing, office equipment, or sitting next to or eating with someone with HIV infection.

Quality Assurance (QA)

External determinants detects errors fixes blame and responsibility post-event investigation QA department is responsible Inspires fear

Emotional abuse

Extreme debasement of a person's feelings so that he or she feels inept, uncared for, and worthless.

A

Factors that contribute to newly emerging or reemerging infectious disease can be related to microbial adaptation and changes made by the infectious agent. However, most of the emergence factors are related to environmental changes and: a. consequences of human activities/behaviors. b. increase in the number of vectors. c. industrialization and urbanization. d. unpredictable variances in the climate.

neglect

Failure to act as an ordinary, prudent person; conduct contrary to that of a reasonable person under a specific circumstance. The failure of a caregiver to provide services that are necessary for the physical and mental health of an individual.

physical neglect

Failure to provide adequate food, proper clothing, shelter, hygiene, or necessary medical care.

Benefits of program planning

Focused on organization and health, reduces role ambiguity, reduces uncertainty, helps to cope, allows for quality decision making

Addiction Treatment

Focuses on the addiction process by helping clients view addiction as a chronic disease and assisting them in making lifestyle changes to halt the progression of the disease.

A

For a nurse to develop a therapeutic attitude toward the treatment of alcohol, tobacco, and other drug (ATOD) problems in the community, the nurse must realize drug addiction can be successfully treated, anyone may develop drug dependence, and: A. any drug can be abused. B. illegal drugs are the category of abused drugs. C. prescription drugs rarely cause dependence. D. over-the-counter (OTC) drugs are "good drugs."

A

For the homeless, health care is usually crisis oriented and sought in emergency departments. The most difficult challenge for nurses treating this vulnerable population is to recognize the client's: A. Limitations in following treatment protocols. B. limited number of transient treatment facilities. C. transition to persistent poverty. D. use and abuse of tobacco, alcohol, and illicit drugs.

B

For the nurse to fully understand the threat associated with the release of biological agents and participate in an appropriate response, the Centers for Disease Control and Prevention (CDC) has stated that the biological agents most likely to be employed are those that both have a potential for high mortality and can be easily disseminated to produce: A. immediately severe symptoms. B. major panic and social disruption. C. multisyndrome effect. D. person-to-person transmission.

Temporary assistance to needy families (TANF)

Formerly called Aid to Families with Dependent Children (AFDC), a federal and state program to provide financial assistance to needy children deprived of parental support because of death, disability, absence from the home, or in some states, unemployment. This program mandates that women heads-of-household find employment to retain their benefits.

Health Professional Shortage Areas (HPSAs)

Geographic areas that have insufficient numbers of health professionals according to criteria established by the federal government. It often consists of rural areas in which a physician, nurse practitioner, or nurse in community health provides services to residents who live in several counties.

Goals of quality assurance

Goals include: to continuously improve the timeliness, effectiveness, safety, and responsiveness of programs to optimize internal resources to improve the health of the community.

Examples of Sexually Transmitted Diseases

Gonorrhea Syphilis Chlamydia Herpes Simplex Virus 2 (Genital Herpes) Human Papillomavirus Infection (HPV) - Can lead to genital warts

value

Ideas of life, customs, and ways of behaving that members of a society regard as desirable

C

Immunity to disease through vaccination is known as: a. natural immunity. b. resistance. c. acquired immunity. d. herd immunity.

The goal of deinstitutionalization was to improve the quality of life for people with mental disorders by providing services in the communities where they lived rather than in large institutions. At what program level did this change in locus of care fail? Assessment level Design level Evaluation level Implementation level

Implementation level. Although deinstitutionalization was noble in conception, it was bankrupt in implementation. Families and communities were not prepared to take on the treatment responsibilities they had to assume, and little education was available. Either care settings such as nursing homes, personal care settings, supervised apartments, rooming houses, single-occupancy hotels, and other similar settings were not available, or people were not educated or prepared to deal with this population.DIF: Cognitive Level: KnowledgeREF: Page 422

A

In addition to those barriers faced by many residents in rural areas, what additional barrier to health care is a Hispanic migrant farmworker likely to encounter? a. Absence of culturally competent care b. Availability of specialists c. Distance of health care facilities from the place of residence d. High cost of health care

C

Indiscriminate use of "good drugs" has caused more health problems from adverse reactions, drug interactions, dependence, addiction, and overdoses than has the use of "bad drugs." The high-risk population group that most experiences the negative consequences identified above is: A. adolescents. B. injection drug users. C. older adults. D. pregnant women.

Chlamydia

Infects the genitourinary tract and rectum of adults and causes conjunctivitis and pneumonia in neonates Transmission occurs when mucopurulent discharge from infected sites, such as the cervix or urethra, comes into contact with the mucous membranes of a non-infected person. Most common reportable infectious disease If left untreated, chlamydia can result in pelvic inflammatory disease (PID).

Chemical Terrorism

Intentional release of hazardous chemicals into the environment for the purpose of harming or killing.

Biological Terrorism

Intentional release of viruses, bacteria, or their toxins for the purpose of harming or killing.

CD

It is estimated that one in 15 males becomes a father during his teen years. The nurse should be aware that many young men facing paternity have specific challenges such as: (select all that apply) Select all that apply. A. acting as though they are interested when they are disinterested. B. avoiding prenatal care involvement. C. desiring and needing to be involved with their children. D. being rejected by the young woman's family. E. rejecting their role as a father.

Public health programs are designed with the goal of improving a population's health status. Which of the following apply to public health?

It is health status monitored at the state level. It is an organized community approach.

homicide

Killing of one human being by another.

feminist ethics

Knowledge and critique of classical ethical theories developed by men and women; entails knowledge about the social, cultural, political, economic, environmental, and professional contexts that insidiously and overtly oppress women as individuals, or within a family, group, community, or society

Migrant Health Act

Legislation passed in the US in 1962 that provides support for clinics serving agricultural workers. Grants were given to community-based and state organizations in the US and its territories to enable them to provide culturally sensitive, comprehensive medical services to migrant and seasonal farmworkers and their families. In 2002, 670,000 people received services from the funds from the Migrant Health Act.

BDE

Marijuana (Cannabis sativa or C. indica) is the most widely used illicit drug in the United States. The nurse should be aware that marijuana has which of the following characteristics? Select all that apply. A. Decreases appetite B. May lead to tolerance C. Is highly toxic D. Has little quality control E. Is a safe therapeutic agent

Marijuana (Cannabis sativa or C. indica) is the most widely used illicit drug in the United States. The nurse should be aware that marijuana has which of the following characteristics? Select all that apply. Decreases appetite May lead to tolerance Is highly toxic Has little quality control Is a safe therapeutic agent

May lead to tolerance. Has little quality control. Is a safe therapeutic agent. Compared with the other psychoactive drugs, marijuana has little toxicity and is one of the safest therapeutic agents known. However, because of its illegal status, there is little quality control, and a user may consume contaminated marijuana that may cause problems. Tolerance can develop, as well as physical dependence; however, withdrawal is benign.DIF: Cognitive Level: KnowledgeREF: Page 439

Methadone maintenance for heroin addiction is a harm reduction strategy because it reduces deviant behavior and introduces addicted persons to the health care system. What is a disadvantage to the use of Methadone? Methadone is inexpensive. Methadone carries a risk of overdose. Methadone prevents relapse. Methadone is long acting and effective orally.

Methadone carries a risk of overdose. Methadone maintenance is a harm reduction intervention because it reduces deviant behavior (needle-sharing practices) and introduces addicted persons to the health care system. Methadone, when administered in moderate or high daily doses, produces a cross-tolerance to other narcotics, thereby blocking their effects and decreasing the craving for heroin. The advantages of methadone are that it is long-acting, effective orally, and inexpensive with few known side effects. However, there is a risk of overdose with the use of Methadone.DIF: Cognitive Level: KnowledgeREF: Page 447

B

Methadone maintenance for heroin addiction is a harm reduction strategy because it reduces deviant behavior and introduces addicted persons to the health care system. What is a disadvantage to the use of Methadone? A. Methadone is inexpensive. B. Methadone carries a risk of overdose. C. Methadone prevents relapse. D. Methadone is long acting and effective orally.

Evaluation

Methods to use to determine whether a service is needed and likely to be used

A nurse in community health is participating in a community service board strategic team that is currently assessing the community's strengths, the local public mental health system, the community's mental health status, and other variables. This best describes what strategic program planning model used today in the public health arena?

Mobilizing for Action Through Planning and Partnership (MAPP)

MAPP

Mobilizing for action through planning and partnership

ethical issues

Moral challenges facing the nursing profession

code of ethics

Moral standards that specify a profession's values, goals, and obligations

Negative Posttest Counseling

Negative: counsel on risk-reduction activities; make sure client understands test may not be truly negative (6-12 weeks before evidence of HIV antibody)

Herpes Simplex Virus 2 (Genital Herpes)

No cure Transmitted through direct exposure and infects the genitalia and surrounding skin Occurs in 20% of American adolescents and adults Linked with development of cervical cancer

violence

Nonaccidental acts, interpersonal or intrapersonal, that result in physical or psychological injury to one or more of the people involved.

educator

Nurse who provides information to clients or staff for the purpose of facilitating learning

sexual assault nurse examiner

Nurses trained in sexual assault examination who perform the physical examination in the emergency department to gather evidence (e.g., hair samples, skin fragments beneath the victim's fingernails, evidence from pelvic examinations using colposcopy) for criminal prosecution of sexual assault.

forensic nursing

Nursing care individuals and communities receive in response to injury in situations where health and law intersect.

Event

Occurrence of a phenomenon of health that can be discretely characterized; it can be environmental, occupational, or biological, naturally occurring, or person induced.

HIV antibody test

Offered in many locations Screening may be confidential or anonymous

Alcohol

Oldest and most widely used psychoactive drug in the world; also known as ethyl alcohol or ethanol.

emotional neglect

Omission of the basic nurturing, acceptance, and caring essential for healthy personal development.

physical abuse

One or more episodes of physical aggression, often resulting in serious physical damage to the internal organs, bones, central nervous system, or sense organs.

advocate

One who works to protect the rights of the client while supporting the client's responsibility for self-determination. Nurses may function as advocates for vulnerable populations by working for the passage and implementation of policies that will result in improved public health services for these populations. An example would be a nurse who serves on a local coalition for uninsured people and works toward development of a plan for sharing the provision of free or low-cost health care by local health care organizations and providers.

Disease Surveillance

Ongoing systematic collection, analysis, interpretation, and dissemination of specific health data for use in public health.

diaster responders

People who work as members of a team in a disaster to feed back information to relief workers to facilitate rapid rescue and recovery

Migrant Farmworker

Person whose primary employment is in agriculture on a seasonal basis, who has been employed in that work within the past 2 years, and who has a temporary abode.

Withdrawal

Physical and psychological symptoms that occur when a drug upon which a person is dependent is removed.

Child neglect

Physical or emotional neglect. Physical neglect refers to the failure to provide adequate food, clothing, shelter, hygiene, or necessary medical care; emotional neglect refers to the omission of basic nurturing, acceptance, and caring essential for healthy personal development.

spouse abuse

Physical, emotional, or sexual mistreatment of a partner or former partner.

Drug dependence

Physiological change in the central nervous system as a result of chronic drug use.

Sources of program evaluation

Population clients (attitude scales) Program records (clinical records) Epidemiological data (morbidity and mortality data)

Positive Posttest Counseling

Positive: counsel about the need for reducing his or her risks and notifying past partners

Level of Prevention--malaria chemoprophylaxis

Primary Prevention

Level of Prevention--responsible sexual behavior

Primary Prevention

Level of Prevention-blood borne pathogen regulations

Primary Prevention

Level of Prevention-federal regulations protecting American cattle from exposure to bovine spongiform encephalopathy

Primary Prevention

Level of Prevention-following childhood immunization recommendations and "no shots no schools" laws

Primary Prevention

Level of Prevention-rabies preexposure immunization

Primary Prevention

Level of Prevention-regulted and inspected municipal water supply

Primary Prevention

Level of Prevention-repellants for preventing vector-borne disease

Primary Prevention

Level of Prevention-restaurant inspections

Primary Prevention

Level of Prevention-safe food handling in the home

Primary Prevention

Level of Prevention-tetanus boosters, flu shots

Primary Prevention

Denial

Primary symptom of addiction. The person may lie about use, play down use, and blame; may also use anger or humor to avoid acknowledging the problem to self and to others.

nonmaleficence

Principle, according to Hippocrates, that requires that we do no harm. It may be impossible to avoid harm entirely, but this principle requires that health care professionals act according to the standards of due care and try to cause the least amount of harm possible

Stewart B. McKinney Homeless Assistance Act of 1994

Public Law 100-77 passed in 1987 officially involved the federal government in meeting the needs of homeless persons. It was intended to respond to the range of emergency needs facing homeless Americans, such as food, shelter, and health care.

ce

Public health programs are designed with the goal of improving a population's health status. Which of the following apply to public health? Select all that apply. a. It is a branch of medicine. b. It is funded at a local level. c. It is health status monitored at the state level. d. It is implemented at the federal level. e. It is an organized community approach.

ethical dilemmas

Puzzling moral problems in which a person, group, or community can envision morally justified reasons for both taking and not taking a certain course of action

cultural accommodation

RN supports and facilitates pts in their use of cultural practices when such practices ARE NOT harmful to the pt; Ex: diet modification-use this instead of that

cultural desire

RN's intrinsic motivation to provide culturally competent care because they to to; cannot be directly taught

Nonfarm Residency

Residence within an area zoned as "city limits"

Human Papillomavirus (HPV) Infection

Results in genital warts Can infect the mouth, genitals, and anus Transmission of HPV occurs through direct contact with warts that result from HPV. Specific types cause cervical cancer. - Gardasil vaccine

BD

Rural residents appear to have a more persistent, endemic level of depression. The factors that may contribute to this level of depression may be related to which of the following? Select all that apply. A. Crisis intervention B. Gaps in continuum of mental health services C. Sufficient number of mental health services D. Tolerance for destructive coping behaviors E. Trust in the health care professionals

Level of Prevention-human immunodeficiency virus testing and treatment

Secondary Prevention

Level of Prevention-immnoglobulin after hep A exposure

Secondary Prevention

Level of Prevention-immunization and chemoprophylaxis as appropriate in meningococcal outbreak

Secondary Prevention

Level of Prevention-quarantine

Secondary Prevention

MDMA (ecstasy)

Semisynthetic drug classified as a mood elevator that produces feelings of empathy, openness, and well-being.

Tuberculosis

Skin testing with purified protein derivative (PPD) Followed by chest radiography for persons with a positive skin reaction and pulmonary symptoms The emergence of multidrug-resistant TB has prompted the use of directly observed therapy (DOT) in the U.S. and other countries to ensure adherence with drug treatment regimens

Mainstream smoke

Smoke inhaled and exhaled by the smoker.

Women, Infants, and Children (WIC)

Special supplemental food program administered by the Department of Agriculture through the state health departments; it provides nutritious foods that add to the diets of pregnant and nursing women, infants, and children younger than 5 years. Eligibility is based on income and nutritional risk as determined by a health professional.

D

Students at first-day orientation at the local community college are discussing alcohol and drug abuse with their freshmen advisor. During a break, some students go outside for soft drinks and snacks. Soon after, the advisor walks by and notices several of these students smoking cigarettes. Based on this, which of the following topics of the drug and alcohol orientation would be most important for the advisor to reemphasize after the break? a. A discussion of which drugs are commonly used on campus b. How students can learn to "just say no" when offered drugs or drinks c. How to recognize and overcome peer pressure to continue bad habits d.The fact that tobacco smoking causes more deaths than any other behavior in the United States

Inhalants

Substances, often common household chemicals, that are inhaled by drug users. Inhalants fall into four categories: volatile organic solvents, aerosols, volatile nitrites, and gases; they are inhaled from bottles, aerosol cans, or soaked cloth.

Sexual victimization

Suffering from a destructive or injurious sexual action.

Sentinel

Surveillance system that monitors key health events when information is not otherwise available or in vulnerable populations to calculate or estimate disease morbidity.

Level of Prevention-Regular inspection of hands and feet, as well as using protective footwear and gloves to avoid trauma and infection for leprosy clients who have lost sensation in those areas

Tertiary Prevention

d

The broad practice of public health requires a set of skills, knowledge, and attitudes in all disciplines. The baccalaureate level of entry for nursing is currently endorsed to achieve the core competencies of public health because of: a. core public health competencies typically being learned in the workplace. b. encouragement for nurses with an associate's degree to seek further degrees. c. financial planning and management skills learned at the master's level. d. rapid change and limited resources in public health.

BC

The community health nurse performs an assessment of violence by observing which of the following community characteristics? Select all that apply. A. Presence of social support networks B. Crime rates C. Levels of unemployment D. Presence of physical disabilities in individuals E. Presence of family violence

C

The local hospital emergency department has recently experienced an increase in gastroenteritis cases among migrant farmworkers. The local health department is informed of this rise in cases and schedules a case mapping of local: A. bars frequented by migrant workers. B. farm fields employing migrant workers. C. housing for migrant workers. D. restaurants frequented by migrant workers.

c

The nurse in community health prepares a community assessment for the local planning commission and presents data that indicate an increase in violence among young children and teens within the community. The nurse projects the cost of care related to violence in ED visits, treatment of stabbing and gunshot wounds, and rehabilitation. The nurse also produces information on school absenteeism, graduation rates, and teenage rape and pregnancy. The planning commission creates a task force to explore the community outlets for child and teen recreation and the current community education programs available to families and students related to violence prevention, negotiation, and mediation. This can best be described as an example of: a. disparities in health care. b. outreach work. c. power of partnerships. d. social issues.

b

The nurse in community health serves as a bridge between the at-risk populations and the community's health care resources. This role is based on the nurse's responsibility to: a. collect and analyze data on programs. b. ensure that all populations have access to affordable, quality health care. c. monitor and assess critical health status indicators. d. provide evidence-based use of resources.

A

The nurse is evaluating a new home health client for ongoing management at home following back surgery for a traumatic injury. The client has been receiving a morphine-based drug for long-term pain management over the past 6 months. The nurse's assessment should include a plan for addressing the client's: A. drug dependence. B. drug addiction. C. substance abuse. D. opiate addiction.

B

The nurse is making a postpartum visit to a teenage mother and her month-old infant. Which of the following assessments would be most important? a. Assessing for mother's ability to fulfill her own growth and development tasks b. Assessing for mother's knowledge about normal infant growth and development c. Assessing how much the teenage mother's own mother is helping her cope with child care d. Assessing whether the baby's father is being helpful to the mother

C

The nurse must ask about violence at each prenatal and postpartum visit, especially with vulnerable populations such as teenagers, as well as observe for signs of violence on each visit. The nurse should be aware that the peak for postpartum intimate partner violence may be observed at various times according to ethnic group, such as: A. 6 months for white mothers. B. 10 months for white mothers. C. 3 months for African-American and Hispanic/Latino mothers. D. 12 months for African-American and Hispanic/Latino mothers.

B

The nurse who works at a homeless clinic wants to improve healing of chronic wounds in clients living on the streets. Which of the following would be the best action to take to improve client outcomes? a. Provide antibiotics to all homeless persons with chronic, nonhealing wounds b. Offer daily access to a room with soap, water, and bandages c. Hand out free bandaging supplies following each clinic visit d. Perform regular monitoring of the client's wound condition

A

The public health administrator is in the process of hiring a new public health nurse. Which of the following statements made by an applicant would most likely result in the applicant not being hired? A. "I like to be the only person working on a project because individuals in teams have their own ideas and plans and the resulting debate slows progress." B. "I prefer to work in teams because no single person has too much responsibility because the burden is shared." C. "Teamwork is better than work done by individuals because teamwork incorporates different perspectives." D. "Whether teamwork is better than work done by individuals depends on the nature of the work being performed."

c

The role/activity of the nurse in community health that was most important during the disasters of September 11, 2001, and the subsequent anthrax bioterrorism attack involved: a. assessment. b. client-level teaching. c. enhanced surveillance activities. d. free and low-cost immunization.

B

The wife of an abusive husband reports to the nurse that her husband has been increasingly more abusive over the past year. Which of the following should be the first action that is taken by the nurse? a. Arrange for the couple to attend marital counseling. b. Develop a plan for the wife's escape if the violence starts again. c. Teach the client how to avoid initiating her husband's anger. d. Work with the client on development of self-esteem.

epidemiologic triangle

Three factors make up the triangle: host, agent, and environment (with vector in the center)...NOTE: changes in the characteristics of any of the factors may result in disease transmission. (Ex. treat an infection with antibiotics and then the normally occurring organisms in the body may get out of balance and you develop a yeast infection.)

A

Three women were brought to the emergency department by ambulance after a two-car accident at a busy intersection. Which of the following tests should automatically be done under such circumstances? a. Alcohol blood level b. Cardiac workup c. Pregnancy test d. STD tests

c

To achieve the major goals of Healthy People 2020, increasing quality and years of healthy life and eliminating health disparities, the partnership between federal, state, and local agencies is critical, and local community coalitions are encouraged because the: a. federal government is ultimately responsible for the health status of the nation. b. public demands that the government protect the people. c. public health departments do not have the resources to accomplish these goals independently. d. state health department would require a universal approach to achieving objectives.

HIV Test Counseling

To receive test results, assess risk Discuss risk behaviors and how to avoid engaging in them Develop with the client a risk-reduction plan Establish the follow-up appointment and posttest counseling Partner notification and contact tracing

Specific Approaches to Quality Improvement

Total quality management/ Continuous quality improvement, tradition quality assurance, staff review committees, utilization review, risk management Professional review organizations Evaluative studies

A

Universal Precautions is a policy for all health care settings, where potential contact with blood or other body fluids exists, and requires that health care workers always perform hand hygiene and wear gloves, masks, protective clothing, and other indicated personal protective barriers. The underlying strategy for this policy requires that: a. blood and body fluids of all clients be handled as if infected. b. health care workers effectively use hand hygiene. c. health care settings are reservoirs of infection. d. effective infection control surveillance programs are in place.

Sentinel method

Uses outcome measures to evaluate the quality of care; based on epidemiological principles.

B

Violence is a major public health problem in our communities that causes premature mortality and lifelong disability. Violence-related morbidity is a significant factor in: A. community deterioration. B. health care costs. C. juvenile delinquency. D. population density.

National Notifiable Disease Surveillance System (NNDSS)

Voluntary system monitored by the Centers for Disease Control and Prevention that includes 52 infectious diseases or conditions with case definitions that are considered important to the public's health.

BCD

What did events after September 11, 2001, clearly demonstrate (Select all that apply.) A. A strong public health structure ready to respond effectively to a crisis B. Public health's need for additional funding and resources C. The unrecognized importance of nurses in lowering health care costs D. Lack of adequate vaccines to meet community's needs

A

When determining whether a geographic area is rural or urban, the nurse should recognize that: A. rural and urban areas, by relative nature, occur on a continuum. B. rural regions have fewer than six persons per square mile. C. rural residents feel isolated. D. rural areas are recreational, retirement, or resort communities.

B

When using the health measure of death rates for working-age adults, the nurse could expect to find the highest rates in which areas? A. Large metropolitan areas B. Most rural and highly populated urban areas C. Most rural and suburban areas D. Small suburban and all urban areas

B

Which of the following addictive drugs is both legal and culturally acceptable throughout almost all of the United States? a. Beer b. Caffeine c. Cigarettes (nicotine) d. Mixed alcoholic beverages

C

Which of the following best describes a health professional shortage area (HPSA)? a. An area with inadequate health care facilities for residents b. An isolated area of underserved populations within an urban region c. A region with insufficient numbers of health care providers d. A rural region of the United States with a population density of less than 10,000

B

Which of the following best describes a principle of care that the nurse should abide by when working with abusive families? a. Confront the abuser and shame him into penitence and a desire to change. b. Demonstrate respect for all family members, including the abuser. c. Exclude the abuser from family meetings because the abuser is the one creating problems. d. Notify the abuser's clergyman, boss, and others who can exert pressure for change.

C

Which of the following best describes how public health nurses (PHNs) differed from other health care professions in the early twentieth century? A. PHNs contributed without receiving any salary for doing so. B. PHNs created new settings where health care could be given. C. PHNs gave care to powerless groups. D. PHNs worked outside any organized health care system.

B

Which of the following best describes the current goal in relation to communicable diseases? a. To control political borders so diseases cannot spread further b. To exterminate specific infectious agents one by one c. To expand health care facilities to improve infectious disease treatment d. To achieve worldwide immunization to control new cases

D

Which of the following best describes the primary reason life spans have notably lengthened over the past 100 years? A. Advances in hospital care and procedures B. Advances in medicine and medical research C. Advances in nursing practice D. Advances in public health

A

Which of the following best describes the role of nursing in community health? A. Application of nursing and public health theory in promoting, preserving, and maintaining health of individuals, families, or communities B. Health care provision offered in primary and secondary institutions or in the client's home C. Provision of health care services in institutions located in the community but outside the hospital D. Use of the nursing process and evidence-based practice to meet Healthy People 2020 objectives for community health improvement

AD

Which of the following best describes why health professionals would be particularly concerned about the health needs of residents in rural areas? (Select all that apply.) a. About 25% of all U.S. residents live in rural settings. b. People in rural areas are especially susceptible to acute illnesses rather than chronic diseases. c. Diagnoses in rural areas are usually for physical injuries, not mental health concerns. d. A high prevalence of poverty exists among rural families.

A

Which of the following best represents an example of infectious disease spreading via a vector? a. Being bitten by an infected mosquito b. Disease spreading from infected mother to infant via the placenta c. A group of partygoers hugging and shaking hands d. Two persons, one of whom is infected, sharing a glass of soda

A

Which of the following biological warfare agents poses the greatest bioterrorism threat to a community? a. Anthrax b. Botulism c. Smallpox d. Tularemia

B

Which of the following components of the epidemiologic triangle contributes most to a female client developing a vaginal infection caused by fungi after successful treatment of her strep throat with antibiotics? a. Agent b. Environment c. Host d. Agent and host

B

Which of the following conditions of the rural environment provides increased opportunities for teaching? a. Increased interaction among residents due to neighbors visiting neighbors on the family farms b. Involvement in rural community activities provides more contact with community residents than in urban areas c. Nursing responsibilities in these areas stress the importance of primary, secondary, and tertiary prevention d. Increased illnesses and injuries of rural residents require that they see nurses more often

C

Which of the following data would most likely be collected in a syndromic surveillance system? a. Incidence of bioterrorism attacks b. Number of air travelers c. Incidence of school absenteeism d. Number of influenza vaccines administered

A

Which of the following factors is causing nursing to change so quickly? A. Economic issues B. Increases in medical and nursing knowledge C. Legislative issues D. Technology changes

BCD

Which of the following factors may lead to increased violence? (Select all that apply.) a. Adequate social support b. Feelings of powerlessness c. Violence shown in the media d. Living in a crowded environment

C

Which of the following federal agencies is most influential in public health activities? A. The Agency for Healthcare, Research, and Quality B. Centers for Disease Control and Prevention C. Department of Health and Human Services D. Food and Drug Administration

B

Which of the following groups is most at risk for suicide? a. Adolescents under age 20 b. Men over age 85 c. Females 25 to 45 years of age d. Women over age 65

D

Which of the following infectious disease interventions best represents the use of secondary prevention? a. Clients with HIV infection are encouraged to use condoms to protect sexual partners. b. Clients with HIV infection are given medications to improve immunological response. c. Health care workers are encouraged to receive annual vaccinations for influenza. d. Health care workers are required to have a tuberculosis skin test or chest x-ray.

C

Which of the following is a common factor typically found in those who abuse others? a. Fear and belief that others are "out to get them" b. Poor self-esteem and unawareness of alternatives c. Previous observations of violent behaviors to manage conflict d. Difficulty keeping commitments

D

Which of the following is a core competency required of public health nurses? A. Knowledge in the use of high-technology diagnostics B. Knowledge of the implementation of electronic medical records C. Skill in the physical assessment of complex clients D. Skill in developing policy and planning programs

B

Which of the following is an accurate concern about the use of marijuana? a. Marijuana has painful withdrawal symptoms. b. Marijuana may damage the respiratory tract. c. Marijuana can often reduce pain but physicians refuse to prescribe it. d. Marijuana quickly leads to psychological and physiological dependence.

C

Which of the following is the leading cause of disability for Americans 15 to 44 years of age? a. Accidents b. Arthritis c. Major depressive disorder d. Workplace injuries

C

Which of the following is the most accurate description of a migrant farmworker? a. A person who does farm work as the primary means of employment, although other work may be done when the seasonal work ends b. A person who immigrates to the United States to "follow the crops" in performing seasonal farm work c. A person who moves from place to place to earn money performing seasonal agricultural work d. A person who specializes in the development of rural land for the purpose of farming

C

Which of the following is the most common vector-borne disease in the United States? a. Babesiosis b. Ehrlichiosis c. Lyme disease d. Rocky Mountain spotted fever

B

Which of the following is the most common vector-borne disease worldwide? a. Dengue b. Malaria c. Onchocerciasis (river blindness) d. Yellow fever

B

Which of the following is the most likely reason for a man to rape a woman? a. Because the woman was asking for it b. For power and control c. Provoked by the woman's dress and behavior d. Sexual pleasure and release

A

Which of the following is the most probable cause of the increase in new emerging infectious diseases? a. Activities or behavior of humans, including changes in the environment b. Increasing urbanization and growth in new housing materials c. New infectious agents are evolving throughout the world d. Overpopulation in many areas, creating a need to reduce global population

B

Which of the following is the most rapidly growing group of homeless? a. Adolescents b. Families with children c. Persons in crisis d. Single men

B

Which of the following is the number one cause of death worldwide? a. Chronic diseases (heart disease, cancer, stroke) b. Infectious diseases c. Injuries (accidental or purposeful) d. Terrorism

B

Which of the following is the underlying science or area of study on which public health is based? A. Biostatistics B. Epidemiology C. Medicine D. Social sciences

A

Which of the following persons is most likely to be assaulted? a. A young man b. A young woman c. An older man d. An older woman

A

Which of the following places best describes where the incidence of Vancomycin-resistant Staphylococcus aureus (VRSA) and methicillin-resistant S. aureus (MRSA) is currently rising? a. Areas where people share dressing or bathing facilities b. Daycare centers and schools c. Long-term care facilities d. Senior citizen centers

C

Which of the following public health actions has been particularly instrumental in reducing childhood infectious diseases in the United States? a. Answering parents' questions about the safety and importance of vaccines today b. Educational campaigns to all health care providers about the importance of immunizations whenever a child is seen c. "No shots, no school" legislation, which legally requires children be immunized before school d. Offering all immunizations to all children free of any charge

B

Which of the following statements accurately describes why methadone is used to treat heroin addiction? a. Addicts come to prefer methadone instead of heroin. b. Methadone blocks the effects of heroin and reduces the craving. c. Methadone prevents further dependence on drugs. d. Methadone gives an enjoyable high, so addicts continue treatment.

D

Which of the following statements best describes why health care of the homeless is so expensive to the community? a. Health conditions of the homeless require increased preventive services. b. Homeless clients typically make more clinic visits for multiple health problems. c. Homeless people spread contagious diseases to those they pass on the street. d. Most care to homeless people takes place in hospital emergency departments.

C

Which of the following statements best explains the primary reason why violence is so high among young African-American men? a. Men tend to be more violent than women. b. They are young and thus lack maturity. c. Unemployment is higher in this population. d. Violent tendencies are transferred genetically along racial lines.

B

Which of the following statements by a parent indicates a need for increased learning regarding appropriate (nonabusive) discipline of a child? a. "I have stopped using a switch on my child, and I am learning to count to 10 before reacting." b. "I never spank or hit; I yell at them to stop being stupid, and if they don't, I tell them no one is ever going to love them if they act like that." c. "I use time-outs when my child acts out or is naughty. Sometimes my child doesn't cope well with this, but I am persistent." d. "When my child misbehaves, I distract him and focus his attention on other things. If he throws a tantrum, I have been known to just pick him up and leave from wherever we are."

D

Which of the following statements is the best explanation as to why some countries are more violent than others? a. Civilized societies are less violent than primitive societies. b. Geographical differences are the primary factor. c. Population differences are the main determinant—for example, having more males than females. d. Violence is a learned behavior controlled or allowed by social norms.

A

Which of the following statements, made by a caregiver of an older client, should alert the nurse to assess for evidence of elder abuse? a. "Mom is always into something and can't seem to stay still, so I've been giving her half a Valium to get her to relax so I can get some rest." b. "Mom wanted to stay at her home, but we were scared for her safety, so we moved some of her personal things into our home and brought her to live with us." c. "She has not been having incontinence problems since we have been taking her to the toilet every 2 to 3 hours when she is awake." d. "We have to feed Mom baby food now because she has trouble chewing and swallowing regular food."

host

a human or animal that harbors an infectious agent

A 3-year smoking cessation program for teens has just concluded. The type of evaluation the staff will conduct is:

a summative evaluation.

Accreditation

a voluntary approach to QA, is used for institutions. Evaluates an agency's physical structure, organizations structure, personnel qualification, and the educational qualifications of its staff.

pathogenicity

ability to produce a specific clinical reaction after infection occurs

antigenicity

ability to stimulate an immunological response

In giving care to the survivors of violence, the nurse should demonstrate respect and caring for all family members, insist that safety is the first priority, and demonstrate intolerance for violent behavior. Additionally, the nurse should be: absolutely honest about what will be reported and what the family can expect. authoritarian in approaching the problem. cautious in reporting unconfirmed reports of violence. sincere in concern for the victims.

absolutely honest about what will be reported and what the family can expect. The principles of giving care to families who have experienced violence include the following: intolerance of the violence, respect and caring for all family members, safety as the first priority, absolute honesty, and empowerment. The nurse must use a nurse-family partnership rather than a paternalistic or authoritarian approach.DIF: Cognitive Level: KnowledgeREF: Pages 470-471

As a result of recent social policy changes, public health agencies have shifted their emphasis from a focus on primary health care services to a focus on core public health activities because of new and re-emerging public health issues. A critical aspect of ensuring the public health system's ability to address these issues is:

adequate funding.

cultural brokering

advocating, mediating, negotiating, and intervening between the pt's culture and the biomedical health care culture on behalf of the pts; similar to Standard 6: Advocacy and Empowerment

Total Quality Management (TQM)

an approach to managing quality of care through appraisal of the nursing process after the client's discharge from the health care system

For a nurse to develop a therapeutic attitude toward the treatment of alcohol, tobacco, and other drug (ATOD) problems in the community, the nurse must realize drug addiction can be successfully treated, anyone may develop drug dependence, and: any drug can be abused. illegal drugs are the category of abused drugs. prescription drugs rarely cause dependence. over-the-counter (OTC) drugs are "good drugs."

any drug can be abused. A health care approach to ATOD problems is the harm reduction model. This is a new public health model that nurses can use to treat individuals, families, and communities. To develop a therapeutic attitude, the nurse must realize that any drug can be abused, anyone may develop drug dependence, and drug addiction can be successfully treated.DIF: Cognitive Level: KnowledgeREF: Page 434

A nurse in community health is working with a parent whose spouse has been called up for active duty in the military reserve. The family is experiencing financial strain due to decreased income. The extended family lives at a distance. The parent is struggling to manage the family in the spouse's absence. The family consists of four children (three preschool and one preteen). In this situation, it would be important for the nurse to further explore the potential for: Child abuse depression intimate partner abuse parent's resentment of the preteen

child abuse. To help abusive families, nurses need to understand that the factors that characterize people who become involved in family violence include upbringing, living conditions, and increased stress. Of these factors, the one most predictably present is previous exposure to some form of violence. As children, abusers were often beaten or saw siblings or parents beaten. They learned that violence is a way to manage conflict. Both men and women who witnessed abuse as children were more likely to abuse their children. Financial solvency and support tended to decrease the incidence of child abuse.DIF: Cognitive Level: ApplicationREF: Pages 460-461

cultural competence

combination of culturally congruent behaviors, practice attitudes, and policies that allow nurses to work effectively in cross-cultural situations

Certifications

combines features of licensure and accreditations.

A nurse in community health seeks a low-cost evaluation method to learn the perspectives of the largest number of persons regarding a proposed local safe haven program for unwanted infants. The best evaluation method to meet the criteria would be:

community forums.

The major sources of information for program evaluation are: (select all that apply)

community indicators. program clients. program records.

A government agency is conducting an audit of all active clients in the local hospice program to ensure that hospice criteria for care are being applied appropriately. The type of tool being used in this specific approach to quality assurance is:

concurrent audit.

The nurse engaging in formative program evaluation would most likely:

conduct medical record audits for quality assurance.

One specific approach to quality assurance is the use of Total Quality Management (TQM). A district public health department uses this approach and gives much attention to ensuring that studies are used to improve processes, remove management by objectives, and promote self-improvement. The major TQM guideline that would summarize these efforts would be:

creating a strong customer-oriented philosophy that is process-driven.

Nurses in community health who understand the basis of their own behaviors and how those behaviors help or hinder the delivery of competent care to persons from cultures other than their own are demonstrating the cultural competence development process construct of:

cultural awareness.

public health nurses

deliver services within the framework of ever-constricting resources coupled with emerging and complex public health issues. This requires the efficient, equitable, and evidence-based use of resources.

A client is back for his follow-up appointment and says to the nurse, "I know. I know. I drink too much, but the job is so stressful that I need to find a way to unwind at the end of the day. You would too!" The nurse should recognize that this is a primary symptom of addiction known as: denial. social drug use patterns. setting variable. coping.

denial. The role of the nurse in secondary prevention is to accurately assess the client to identify substance abuse and plan appropriate interventions. A progression in drug-use patterns and related problems warns about the possibility of addiction. Denial is a primary symptom of addiction and can be demonstrated by the following: lying about use; minimizing use patterns; blaming or rationalizing; intellectualizing; changing the subject; using anger or humor; and "going with the flow" (agreeing that a problem exists, stating behavior will change, but not demonstrating any behavior change).DIF: Cognitive Level: ApplicationREF: Page 443

It is estimated that one in 15 males becomes a father during his teen years. The nurse should be aware that many young men facing paternity have specific challenges such as: (select all that apply) Select all that apply. acting as though they are interested when they are disinterested. avoiding prenatal care involvement. desiring and needing to be involved with their children. being rejected by the young woman's family. rejecting their role as a father.

desiring and needing to be involved with their children. being rejected by the young woman's family. Nurses can acknowledge and support the young man as he develops in the role of father. Young men react differently when they learn that their partner is pregnant, and it often depends on the nature of the relationship before pregnancy. Many young men will accompany the young woman to a health care center for pregnancy diagnosis and counseling. A large percentage of young men will continue to accompany the young woman to some prenatal visits and may even attend the delivery. These young men may also want to and need to be involved with their children regardless of changes in the relationship with the teen mother. It is not unusual for a young man to be excluded or even rejected by the young woman's family. He may then begin to act as if he is disinterested when he may really feel that he cannot provide resources for his child or know how to take care of the child.DIF: Cognitive Level: ComprehensionREF: Pages 417-418

A case management nurse for a locally funded program for special-needs children is increasingly concerned about a recent referral for a 1-year-old child with a congenital illness residing in a poverty-stricken community. The nurse knows that this child may be at higher risk for the most harmful effects of poverty, including: developmental delays. ear infections. frequent colds and infections. irritability.

developmental delays. Young children (newborn to age 5 years) are at greater risk for the most harmful effects of poverty, especially in regard to adequate nutrition and brain development. Other risk factors include maternal substance abuse or depression, exposure to environmental toxins, trauma and abuse, and poor-quality daily care.DIF: Cognitive Level: ApplicationREF: Page 411

The nurse at the adult day care center notices bruises on the wrists of a 90-year-old client. Besides the physical assessment of the client, the nurse should: confront the daughter when she arrives to pick the father up. discuss the findings with the caregivers to determine the cause of the injuries. educate the staff about indications of elder abuse. make a referral to the primary care provider for follow-up.

discuss the findings with the caregivers to determine the cause of the injuries. Rough handling by caregivers can lead to bruises and bleeding into body tissues because of the fragility of older adult clients' skin and vascular systems. It is often difficult to determine whether the injuries of older adults result from abuse, falls, or other natural causes. Careful assessment through both observation and discussion can help determine the cause of injuries so that proper plans for interventions can be made.DIF: Cognitive Level: KnowledgeREF: Page 467

communicable diseases

diseases of human or animal origin caused by an infectious agent and resulting from transmission of that agent from an infected person, animal, or inanimate source to a susceptible host. Not all communicable diseases are communicated from host to host. (Ex. tetanus is transmitted from an inanimate source to a person but then cannot be passed from the infected person to another person.)

The nurse is evaluating a new home health client for ongoing management at home following back surgery for a traumatic injury. The client has been receiving a morphine-based drug for long-term pain management over the past 6 months. The nurse's assessment should include a plan for addressing the client's: drug dependence. drug addiction. substance abuse. opiate addiction.

drug dependence. Drug dependence is a state of neuroadaptation caused by the chronic, regular administration of a drug. This is a physiological change in the central nervous system; therefore the drug must be continued to prevent withdrawal symptoms. The morphine should be gradually tapered rather than abruptly stopped to prevent withdrawal symptoms.DIF: Cognitive Level: ApplicationREF: Page 435

Local officials have requested a program evaluation of a comprehensive teen sex education program offered in the local schools in preparation for potential budget discussions. A nurse in community health conducts a program evaluation and determines that the teen pregnancy rate has gradually declined over the years that the program has been in place. The community is measuring the program's:

efficiency.

The role/activity of the nurse in community health that was most important during the disasters of September 11, 2001, and the subsequent anthrax bioterrorism attack involved:

enhanced surveillance activities.

The nurse in community health serves as a bridge between the at-risk populations and the community's health care resources. This role is based on the nurse's responsibility to:

ensure that all populations have access to affordable, quality health care.

A Public Health Service nurse working for the Indian Health Service is working with a client diagnosed with cancer. The client uses sweat lodges to "cure the disease." The nurse understands that within the culture, disease is often perceived as disharmony with other forces, and clients may look to hot or cold treatments to resolve or cure a cancerous condition. The nurse is integrating her knowledge of the cultural organizing factor of:

environmental control.

A major evaluation source for a nurse to use to determine the effectiveness of a teen driver safety program is:

epidemiologic data.

A father brings his stepdaughter to the family clinic for an immunization update before the new school year. The nurse notices the interaction between the young girl and her stepfather. The child appears tense and cautious and wraps her arms around herself in a protective manner. The child startles when touched by the stepfather and pulls away. The stepfather is overheard saying, "I'll leave you here if you don't behave and act nice." In this scenario, it would be important for the nurse to explore in her assessment the possibility of: child neglect. family secrets. father-daughter incest. impaired family functioning.

father-daughter incest. Incest occurs in all races, religious groups, and socioeconomic classes. A typical pattern is as follows: The daughter involved in the parental incest is usually 9 years of age at the onset and is often the oldest or only daughter. The father seldom uses force. He is more likely to use threats, bribes, intimidations, or misrepresentation of moral standards. These children may have difficulty in social situations and demonstrate avoidance behaviors. They may also attempt to cover or protect their bodies. Therefore the nurse must be aware of these indicators in order to conduct an appropriate assessment and plan appropriate interventions.DIF: Cognitive Level: ApplicationREF: Page 465

All adults should be assessed for violence in their primary intimate relationships. The abuse of female partners has the most serious community health ramifications because of the greater prevalence, the more serious long-term emotional and physical consequences, and the greater potential for: fleeing to a shelter. homicide. possessive behavior. spontaneous abortion.

homicide. The abuse of female partners has the most serious community health ramifications because of the greater prevalence, the greater potential for homicide, the effects on the children in the household, and the more serious long-term emotional and physical consequences. As a woman tries to leave the abusive relationship, the risk for homicide increases, creating a catch-22 scenario. A nurse encountering evidence of severe abuse needs to consider the safety of the woman and her children as the priority.DIF: Cognitive Level: ApplicationREF: Page 465

Quality improvement Goals/outcomes

improving safety of care saves lives Cost reduction by using effective interventions increases in client confidence in health care delivery regardless of setting

The Omaha System is a client classification system developed by the Visiting Nurses Association of Omaha, Nebraska, that has the potential to improve the delivery of care by:

improving the description of care.

A new role for public health that is used in widespread public emergencies is:

incident commander.

Injection Drug users (IDUs)

includes intravenous and subcutaneous drug injection, with the latter usually being over the abdominal area and called "popping." The sharing of paraphernalia to prepare or inject the drug can result in transmission of blood-borne pathogens, such as human immunodeficiency virus (HIV).

Complicated gonorrhea

includes salpingitis, epididymitis, systemic gonococcal infection, and gonococcal meningitis.

health care-acquired infections

infections acquired during hospitalizations or developed within a health care setting: they may involve clients, health care workers, visitors, or anyone who has contact with a health care setting.

An eighth-grader is brought to the emergency department by a parent for unusual skin blistering and discoloration around the nose and lips. The parent states that the child will not say what caused the injury. The nurse should consider the possible use of: crystallized methamphetamine. inhalants. MDMA (Ecstasy). PCP (phencyclidine).

inhalants. Inhalants are often among the first drugs that young children use. The primary abusers of most inhalants are adolescents who are 12 to 17 years of age. Use often ends in late adolescence. The 2008 National Survey on Drug Use and Health found that 729,000 persons age 12 or older had used inhalants for the first time in the past 12 months and 70% of them were under the age of 18. Depending on the dose, the user may feel a slight stimulation, less inhibition, or even lose consciousness. There is a link between school performance and use of an inhalant. Other signs include paint or stains on clothes or the body; spots or sores around the mouth; red or runny eyes or nose; chemical breath odor; a drunk, dazed, or dizzy appearance; nausea and loss of appetite; and finally anxiety, excitability, and irritability.DIF: Cognitive Level: ApplicationREF: Page 440

In applying Ethical Principles for Effective Advocacy, the ethical principle that empowers the client (groups or communities) to make knowledgeable decisions is:

keep the client properly informed.

For the homeless, health care is usually crisis oriented and sought in emergency departments. The most difficult challenge for nurses treating this vulnerable population is to recognize the client's: limitations in following treatment protocols. limited number of transient treatment facilities. transition to persistent poverty. use and abuse of tobacco, alcohol, and illicit drugs.

limitations in following treatment protocols. Homeless people devote a large portion of their time to just trying to survive. Health care is usually crisis oriented and sought in emergency departments. Those who access health care have a hard time following prescribed treatment regimes such as prescribed diets, purchasing prescriptions, or health-promotion or symptom-relief measures.DIF: Cognitive Level: KnowledgeREF: Pages 413-414

A nurse in community health has determined that there is a need for a program for teenage fathers who want to learn about child care. The next step in the program management process would be to:

meet with community members to form a planning body.

A hospice nurse is working with a cancer client and his family. The client's 7-year-old son has developed recent school and peer problems. Understanding the risk for disruption of normal development, the nurse plans to first screen the child for: developmental disorders. mental health problems. parental neglect. violence potential.

mental health problems. Children are at risk for disruption of normal development by biological, environmental, and psychosocial factors that impair their mental health, interfere with their education and social interactions, and keep them from realizing their full potential as adults. Children can develop depression or behavior problems in response to an actual or potential loss.DIF: Cognitive Level: ApplicationREF: Page 422

Quality assurance

monitoring the activities of client care to determine the degree of excellence attained in the implementation of the activities

Ethical decision making is conducted in a generic framework. One step of ethical decision making is to place an ethical issue or dilemma within a meaningful context. The rationale for this step is:

multiple factors affect the formulation and justification of ethical issues and dilemmas.

vertical transmission

occurs when transmission passes from parent to offspring via sperm, placenta, milk, or contact in the vaginal canal at birth....examples include transplancental transmission of HIV and syphilis.

Indiscriminate use of "good drugs" has caused more health problems from adverse reactions, drug interactions, dependence, addiction, and overdoses than has the use of "bad drugs." The high-risk population group that most experiences the negative consequences identified above is: adolescents. injection drug users. older adults. pregnant women.

older adults. Older adults consume more prescribed and OTC medications than does any other age group. Problems with alcohol consumption, including interactions with prescribed and OTC drugs, far outnumber any other substance-abuse problem among older adults. Factors such as slowed metabolic turnover of drugs, age-related organ changes, enhanced drug sensitivities, a tendency to use drugs over longer periods, and a more frequent use of multiple drugs contribute to greater negative consequences from drug use among older adults.DIF: Cognitive Level: ComprehensionREF: Pages 444-445

vaccines

one of the most effective ways of preventing and controlling communicable diseases...they are a preparation of killed microorganisms, living attenuated organisms, or living fully virulent organisms that is administered to produce or artificially increase immunity to a particular disease.

disease

one of the possible outcomes of infection and it may indicate a physiological dysfunction or pathological reaction. (Ex. a person positive for HIV and showing clinical signs of AIDS is both infected and diseased.)

A multidisciplinary quality assurance team has reviewed an organization's stated philosophy and objectives and developed a conceptual model for appraisal that integrates peer review and client satisfaction. The quality assurance program component that should be addressed next would be:

outcome.

A disease outbreak of severe acute respiratory syndrome has occurred and has spread over several countries outside the United States. This level of occurrence is said to be:

pandemic.

Provision 3 of the Code of Ethics for Nurses states, "The nurse promotes, advocates for, and strives to protect the health, safety, and rights of the patient." In applying this provision to practice, the public health nurse understands that:

patient means the recipient of nursing care.

cultural conflict

perceived threat that may arise from a misunderstanding of expectations when either group is not aware of cultural differences

cultural encounter

permits RNs to seek opportunities to engage in cross-cultural interactions; interacting with other cultures

horizontal transmission

person to person spread of infection through one or more of the following four routes: direct or indirect contact, common vehicle, airborne, or vector-borne. Examples—most STDs are are spread by direct contact. Enterobiasis, or pinworm infection, can be acquired through direct contact or indirect contact with contaminated objects such as toys, clothing, and bedding. Included, as well, are bed bugs, which are a growing problem with horizontal transmission.

The most significant commonalities between the ANA's Scope and Standards of Public Health Nursing Practice and the Public Health Code of Ethics assert that advocacy is conducted in the:

policy arena.

The general approach to quality improvement known as licensure that grants control over who can enter into and who exists in a profession can best be described as a contract between the:

profession and the state.

To achieve the major goals of Healthy People 2020, increasing quality and years of healthy life and eliminating health disparities, the partnership between federal, state, and local agencies is critical, and local community coalitions are encouraged because the:

public health departments do not have the resources to accomplish these goals independently.

A state health department wants to ensure that the local health departments are delivering quality client care and can demonstrate the degree of excellence attained. The principle that is best defined by this statement is:

quality assurance (QA).

pandemic

refers to an epidemic that is world-wide and and affects large populations. from the lecture—global disease spread..one case of Ebola in the US made it a pandemic...SARS went from endemic in China to epidemic in the US and that made it also pandemic....the plague

passive immunization

refers to immunization through the transfer of a specific antibody from an immunized individual to a non immunized individual, such as the transfer from mother to baby or by andministraion of an antibody containing preparation (immunoglobulin or antiserum—which is immediate but short lived immunity—examples of immunoglobulins typically given are the ones to Hep A, rabies, and tetanus...they are used until the active immunity from an immunization has time to take effect.

infection

refers to the entry, development, and multiplication of the infectious agent in the susceptible host OR the state produced by the invasion of a host by an infectious agent. Such infection may or may not produce visible signs. (Ex. HIV positive means infection, but if no clinical signs are shown then that person is not diseased)

active immunization

refers to the immunization of an indvidual by administration of an antigen (infectious agent or vaccine) and is usually characterized by the presence of an antibody produced by the individual host. (Ex. are vaccinations of people to induce active immunity.)

epidemic

refers to the occurrence of a disease in a community or region in excess of normal expectancy. From the lecture....greater than the normal rate of infection in a population...no Ebola in the US, so one case constitutes an epidemic...epidemic doesn't necessarily mean a large number of cases...just one case of something considered to have previously been eliminated from that area is an epidemic.

common vehicle

refers to transportation of the infectious agent from an infected host to a susceptible host via food, water, milk, blood, serum, saliva, or plasma. Hep A can be transmitted through contaminated food and water, Hep B can be transmitted through contaminated blood. Legionellosis and TB are both spread via contaminated droplets in the air.

Farm Residency

residency outside the area zoned as "city limits"; usually infers involvement in agriculture.

The purpose of record keeping in public health agencies is to maintain complete information on clients served and the extent and quality of service provided to those clients. The records also provide information for education and research. Another important use of the records is to:

resolve legal issues in malpractice suits.

natural immunity

species-determined, innate resistance to an infectious agent (Ex. opossums rarely get rabies)

Gardasil vaccine for HPV

stimulates the immune system to block HPV infection before it occurs. The recommended age for vaccination in females is 11 to 12 years of age, but it can be given in those 9 to 26 years of age.

Universal Precautions

strategy to prevent exposure to pathogens transmitted through blood and other bodily fluids by requiring blood and body fluids from ALL CLIENTS to be handled as if they were infected with such pathogens

Donabedian's structure-process-outcome model

structure: Evaluating the setting and instruments used to provide care: examples of structure are facilities, equipment, characteristics of the administrative organization, client mix, and the qualification of health providers Process: evaluating activities as they relate to standards and expectations of health providers in the management of client care Outcome: the net change or result that occurs as a result of health care.

A long-distance truck driver being assessed by a nurse in a community-based clinic states, "I smoke 3 packs of cigarettes a day. I use coffee and diet pills from the drug store to stay awake on the road. That makes it difficult to sleep when I do pull over, so I use a prescription sleeping pill from my doctor to sleep for 4 hours. It's giving me palpitations." The nurse's assessment should include a diagnosis of: drug abuse. drug addiction. substance abuse. adverse drug reaction.

substance abuse. Substance abuse is the use of any substance that threatens a person's health or impairs social or economic functioning. This definition is more objective and universal than the government's definition of drug abuse, which is the use of a drug without a prescription or any use of an illegal drug. Drug addiction is a pattern of abuse characterized by an overwhelming preoccupation with the use (compulsive use) of a drug and securing its supply and a high tendency to relapse if the drug is removed. Adverse drug reactions are associated with side effects.DIF: Cognitive Level: ApplicationREF: Page 435

cultural preservation

supports and facilitates the use of scientifically supported cultural practices along with those f/ biomedical health care system; Ex: acupuncture, acupressure, and discussing care with family members

A nurse in community health becomes aware that a teen smoking cessation program offered at the health department is a demonstration project. In evaluating this program, the nurse would be concerned with the program's:

sustainability.

cultural blindness

tendency to ignore all differences among cultures, to act as though these differences do not exist, and as a result to treat all ppl the same

Abortion

termination of a pregnancy by spontaneous or induced expulsion of a human fetus during the first 12 weeks of gestation

resistance

the ability of the host to withstand the infection (it may involve natural or acquired (active or passive) immunity)

acquired immunity

the resistance acquired by a host as a result of previous natural exposure to an infectious agent. Ex. having measles once protest against future infection. NOTE: acquired Immunity may be induced by active or passive immunization)

incubation period

the time interval between invasion by an infectious agent and the first appearance of signs and symptoms of the disease. (Ex. the incubation period can vary from 2-4 hours for some food borne illnesses to as much as 10-15 years for AIDS.)

emerging infectious diseases

those in which the incidence has actually increased in the past two decades or has the potential to increase in the near future.

Given recent vaccine shortages for the flu, the local nurses in community health form a group to evaluate the process of scheduling and operating flu vaccination clinics in the community and review the community's complaints from the previous season. The activity best represents the principle of:

total quality management (TQM) to achieve continuous quality improvement (CQI).

A nurse in community health conducts quarterly mental health-promotion and depression-screening programs at the local senior center. The nurse is aware that older adults are at increased risk for developing depression. Using such an intervention also addresses the older adults': dependence on their primary care provider. normal sensory losses. reduced social contacts. underutilization of the mental health system.

underutilization of the mental health system. Older adults, because they may depend on others for care, are at risk for abuse and neglect. Healthy aging activities such as physical activity and establishing social networks improve the mental health of older adults. Older adults underutilize the mental health system and are more likely to be seen in primary care or to be the recipient of care in institutions. The nurse can reach them by organizing health-promotion programs through senior settings or other community-based settings.DIF: Cognitive Level: ApplicationREF: Page 425


Related study sets

Pythagorean Theorem and Volume Review

View Set

Defining Research with Human Subjects

View Set

Fundamentals of Nursing EAQs Ch. 42 Death and Loss

View Set

ENT 466: FINAL EXAM (CHAPTERS 11-13)

View Set

FUNDAMENTALS OF DIGITAL MARKETING

View Set

Evolve Ch 1: Mental Health and Mental Illness

View Set

Network+ N10-008: Exam Premium - Practice Set B

View Set

Semester 2 Spanish 2 H- Multiple Choice

View Set